fork(1) download
  1. #include <stdio.h>
  2. #include <math.h>
  3.  
  4.  
  5. int a, b;
  6. int mm, nn;
  7. int t;
  8.  
  9. float pi = 3.1415926535897932384626433832795;
  10. float pi7 = 0.4487989505128276054946633404685;
  11. float r7 = 2.6457513110645905905016157536393;
  12.  
  13. float sn2, sn4, sn8;
  14. float cs2, cs4, cs8;
  15. float tn2, tn4, tn8;
  16.  
  17. float alpha, beta, gamma;
  18.  
  19.  
  20. #define MAX 128
  21. typedef char Str[MAX];
  22.  
  23. ///////////////////////////////
  24. int ct = 1;
  25.  
  26.  
  27.  
  28.  
  29. int cvt(float x) {
  30. if (x > 0.0)
  31. return (int)(x+0.9);
  32. if (x < 0.0)
  33. return (int)(x-0.9);
  34.  
  35. return 0;
  36. }
  37.  
  38.  
  39. float xabs(float x) {
  40. if (x < 0.0)
  41. return -x;
  42. return x;
  43. }
  44.  
  45. float croot(float num) {
  46. int flag = 1.0;
  47. float x;
  48. if (num < 0.0) {
  49. flag = -1.0;
  50. num = -num;
  51. }
  52. x = flag * exp(log(num)/3.0);
  53.  
  54. // printf("croot: num = %f, x = %f\n", num, x);
  55.  
  56. return x;
  57. }
  58.  
  59. float xsqrt(float num) {
  60. int flag = 1.0;
  61. float x;
  62. if (num < 0.0) {
  63. flag = -1.0;
  64. num = -num;
  65. }
  66. x = flag * exp(log(num)/2.0);
  67.  
  68. // printf("croot: num = %f, x = %f\n", num, x);
  69.  
  70. return x;
  71. }
  72.  
  73. float pwrf(float f, int p) {
  74. int i, neg;
  75. float x = 1.0;
  76.  
  77. if (p == 0)
  78. return 1.0;
  79.  
  80. neg = 0;
  81. if (p < 0) {
  82. neg = 1;
  83. p = -p;
  84. }
  85.  
  86. for (i=0; i< p; i++)
  87. x = x*f;
  88.  
  89. if (neg == 1)
  90. x = 1.0/x;
  91.  
  92. return x;
  93. }
  94.  
  95. int isInt(float x) {
  96. float y = 1.0*cvt(x);
  97.  
  98. if (xabs(x-y) < 0.01)
  99. return 1;
  100.  
  101. return 0;
  102. }
  103.  
  104. //////////////////////////////
  105. int toInt(float x) {
  106. float y;
  107. int u;
  108.  
  109. if (x > 0.1) {
  110. y = x+0.1;
  111. }
  112. else if (x < 0.1) {
  113. y = x-0.1;
  114. }
  115. else
  116. y = 0.0;
  117.  
  118. u = (int)(y);
  119.  
  120.  
  121.  
  122. return u;
  123. }
  124.  
  125.  
  126.  
  127.  
  128. // p >= 0
  129. int pwri(int f, int p) {
  130. int i;
  131. int x = 1;
  132.  
  133. if (p == 0)
  134. return 1;
  135.  
  136.  
  137. for (i=0; i< p; i++)
  138. x = x*f;
  139.  
  140. return x;
  141. }
  142.  
  143.  
  144. void make_eq(int a, int b, int c, Str eq){
  145.  
  146. sprintf(eq, "x^3");
  147. if (a < -1) {
  148. sprintf(eq, "%s - %d x^2", eq, -a);
  149. }
  150. else if (a == -1) {
  151. sprintf(eq, "%s - x^2", eq);
  152. }
  153. else if (a > 1) {
  154. sprintf(eq, "%s + %d x^2", eq, a);
  155. }
  156. else if (a == 1) {
  157. sprintf(eq, "%s + x^2", eq);
  158. }
  159.  
  160. if (b < -1) {
  161. sprintf(eq, "%s - %d x", eq, -b);
  162. }
  163. else if (b == -1) {
  164. sprintf(eq, "%s - x^2", eq);
  165. }
  166. else if (b > 1) {
  167. sprintf(eq, "%s + %d x^2", eq, b);
  168. }
  169. else if (b == 1) {
  170. sprintf(eq, "%s + x^2", eq);
  171. }
  172.  
  173.  
  174. if (c < -1) {
  175. sprintf(eq, "%s - %d", eq, -c);
  176. }
  177. else if (c == -1) {
  178. sprintf(eq, "%s - 1", eq);
  179. }
  180. else if (c > 1) {
  181. sprintf(eq, "%s + %d", eq, c);
  182. }
  183. else if (c == 1) {
  184. sprintf(eq, "%s + 1", eq);
  185. }
  186.  
  187.  
  188. sprintf(eq, "%s = 0", eq);
  189.  
  190.  
  191. }
  192.  
  193.  
  194. //////////////////////////////////////////////
  195.  
  196. int tt(int k) {
  197. int x;
  198. x = k*k*k - 3*(a+b+3)*k-(a*b + 6*(a + b) + 9);
  199. return x;
  200. }
  201.  
  202.  
  203. void print_abg(){
  204.  
  205. // alpha = pwrf(sn2, m)/(pwrf(sn4, n))*(pwrf(sn8, n-m));
  206.  
  207. if (mm>=nn) {
  208. printf("\\alpha = \\frac{\\sin^{%d}(2\\theta)}{\\sin^{%d}(4\\theta) \\sin^{%d}(8\\theta)},\n", mm, nn, mm-nn);
  209. printf("\\beta = \\frac{\\sin^{%d}(4\\theta)}{\\sin^{%d}(8\\theta) \\sin^{%d}(2\\theta)},\n", mm, nn, mm-nn);
  210. printf("\\gamma = \\frac{\\sin^{%d}(8\\theta)}{\\sin^{%d}(2\\theta) \\sin^{%d}(4\\theta)},\n", mm, nn, mm-nn);
  211. }
  212. else {
  213. printf("\\alpha = \\frac{\\sin^{%d}(2\\theta)\\sin^{%d}(8\\theta)}{\\sin^{%d}(4\\theta) },\n", mm, nn-mm, nn);
  214. printf("\\beta = \\frac{\\sin^{%d}(4\\theta)\\sin^{%d}(2\\theta)}{\\sin^{%d}(2\\theta) },\n", mm, nn-mm, nn);
  215. printf("\\gamma = \\frac{\\sin^{%d}(8\\theta)\\sin^{%d}(4\\theta)}{\\sin^{%d}(8\\theta) },\n", mm, nn-mm, nn);
  216. }
  217. }
  218. void process(int sel) {
  219. int k, t;
  220. int x, y;
  221. Str eq;
  222.  
  223. if (sel == 1) {
  224. if ((a+b+3) == 0) {
  225. x = a*b-9;
  226.  
  227. make_eq(-a, b, -1, eq);
  228.  
  229.  
  230.  
  231. printf("\\begin{theorem} \\label{tmn_%d_%d} \n", mm, nn);
  232. printf("Let \n");
  233. printf("$$\n");
  234. print_abg();
  235. printf("$$\n");
  236.  
  237.  
  238. printf("Then $ \\{ \\alpha, \\beta, \\gamma \\} $ are roots of the equation:\n");
  239.  
  240. printf("\\begin{equation} \\label{emn_%d_%d} \n", mm, nn);
  241. printf("%s\n", eq);
  242. printf("\\end{equation} \n");
  243. printf("which satisfies Rammnujan condition. \n");
  244. printf("And \n");
  245.  
  246. if (x >= 0) {
  247. printf("\\begin{equation} \\label{fmn_%d_%d} \n", mm, nn);
  248. printf("\\sqrt[3]{\\alpha} + \\sqrt[3]{\\beta} + \\sqrt[3]{\\gamma} = \n");
  249. printf("\\sqrt[3]{%d + 3\\sqrt[3]{%d}} \n", a+6, x);
  250. printf("\\end{equation} \n");
  251. printf("\\begin{equation} \\label{gmn_%d_%d} \n", mm, nn);
  252. printf("\\frac{1}{\\sqrt[3]{\\alpha}} + \\frac{1}{\\sqrt[3]{\\beta}} + \\frac{1}{\\sqrt[3]{\\gamma}} = \n");
  253. printf("\\sqrt[3]{%d + 3\\sqrt[3]{%d}} \n", b+6, x);
  254. printf("\\end{equation} \n");
  255. }
  256.  
  257. else {
  258. printf("\\begin{equation} \\label{fmn_%d_%d} \n", mm, nn);
  259. printf("\\sqrt[3]{\\alpha} + \\sqrt[3]{\\beta} + \\sqrt[3]{\\gamma} = \n");
  260. printf("\\sqrt[3]{%d - 3\\sqrt[3]{%d}} \n", a+6, -x);
  261. printf("\\end{equation} \n");
  262. printf("\\begin{equation} \\label{gmn_%d_%d} \n", mm, nn);
  263. printf("\\frac{1}{\\sqrt[3]{\\alpha}} + \\frac{1}{\\sqrt[3]{\\beta}} + \\frac{1}{\\sqrt[3]{\\gamma}} = \n");
  264. printf("\\sqrt[3]{%d - 3\\sqrt[3]{%d}} \n", b+6, -x);
  265. printf("\\end{equation} \n");
  266. }
  267.  
  268. printf("\\end{theorem}\n\n");
  269.  
  270.  
  271. return;
  272. }
  273. }
  274.  
  275. if (sel == 2) {
  276. for (k = -1000; k<= 1000; k++) {
  277. if (tt(k) == 0) {
  278. t = k;
  279.  
  280. // printf("(%3d,%3d}: a = %6d, b = %6d, t = %6d \n", mm, nn, a, b, t);
  281.  
  282. // sum a = \sqrt[3]{a+6+3t},
  283. // sum b = \sqrt[3]{b+6+3t},
  284. x = a+6+3*t;
  285. y = b+6+3*t;
  286.  
  287. make_eq(-a, b, -1, eq);
  288.  
  289. printf("\\begin{theorem} \\label{tmn_%d_%d} \n", mm, nn);
  290. printf("Let \n");
  291. printf("$$\n");
  292. printf("\\alpha = \\frac{(2\\cos(2\\theta)^%d}{(2\\cos(4\\theta)^%d},\n", mm, nn);
  293. printf("\\beta = \\frac{(2\\cos(4\\theta)^%d}{(2\\cos(8\\theta)^%d},\n", mm, nn);
  294. printf("\\gamma = \\frac{(2\\cos(8\\theta)^%d}{(2\\cos(2\\theta)^%d}.\n", mm, nn);
  295. printf("$$\n");
  296.  
  297.  
  298. printf("Then $ \\{ \\alpha, \\beta, \\gamma \\} $ are roots of the equation:\n");
  299.  
  300. printf("\\begin{equation} \\label{emn_%d_%d} \n", mm, nn);
  301. printf("%s\n", eq);
  302. printf("\\end{equation} \n");
  303. printf("The associatd Rammnujan equation has integer solution $ %d $. \n", t);
  304. printf("And \n");
  305.  
  306. if (x >= 0) {
  307. printf("\\begin{equation} \\label{fmn_%d_%d} \n", mm, nn);
  308. printf("\\sqrt[3]{\\alpha} + \\sqrt[3]{\\beta} + \\sqrt[3]{\\gamma} = \n");
  309. printf("\\sqrt[3]{%d} \n", x);
  310. printf("\\end{equation} \n");
  311. }
  312.  
  313. else {
  314. printf("\\begin{equation} \\label{fmn_%d_%d} \n", mm, nn);
  315. printf("\\sqrt[3]{\\alpha} + \\sqrt[3]{\\beta} + \\sqrt[3]{\\gamma} = \n");
  316. printf("-\\sqrt[3]{%d} \n", -x);
  317. printf("\\end{equation} \n");
  318. }
  319.  
  320. if (y >= 0) {
  321. printf("\\begin{equation} \\label{gmn_%d_%d} \n", mm, nn);
  322. printf("\\frac{1}{\\sqrt[3]{\\alpha}} + \\frac{1}{\\sqrt[3]{\\beta}} + \\frac{1}{\\sqrt[3]{\\gamma}} = \n");
  323. printf(" \\sqrt[3]{%d} \n", y);
  324. printf("\\end{equation} \n");
  325. }
  326.  
  327. else {
  328. printf("\\begin{equation} \\label{gmn_%d_%d} \n", mm, nn);
  329. printf("\\frac{1}{\\sqrt[3]{\\alpha}} + \\frac{1}{\\sqrt[3]{\\beta}} + \\frac{1}{\\sqrt[3]{\\gamma}} = \n");
  330. printf("-\\sqrt[3]{%d} \n", -y);
  331. printf("\\end{equation} \n");
  332. }
  333. printf("\\end{theorem}\n\n");
  334.  
  335.  
  336. /*
  337.   printf(" sum a = %f, expect: %f\n",
  338.   croot(1.0*x), croot(alpha) + croot(beta) + croot(gamma));
  339.   printf(" sum b = %f, expect: %f\n",
  340.   croot(1.0*y), croot(1.0/alpha) + croot(1.0/beta) + croot(1.0/gamma));
  341.   if (x >= 0)
  342.   printf(" sum a = \\sqrt[3]{%d} \n", x);
  343.   else
  344.   printf(" sum a = -\\sqrt[3]{%d} \n", -x);
  345.  
  346.   if (y >= 0)
  347.   printf(" sum b = \\sqrt[3]{%d} \n", y);
  348.   else
  349.   printf(" sum b = -\\sqrt[3]{%d} \n", -y);
  350. */
  351.  
  352. return;
  353. }
  354. }
  355. }
  356.  
  357. };
  358.  
  359. ////////////////////////////////////////////////////////
  360.  
  361.  
  362. void init () {
  363. sn2 = sin(2*pi7);
  364. sn4 = sin(4*pi7);
  365. sn8 = sin(8*pi7);
  366.  
  367. cs2 = cos(2*pi7);
  368. cs4 = cos(4*pi7);
  369. cs8 = cos(8*pi7);
  370.  
  371. tn2 = tan(2*pi7);
  372. tn4 = tan(4*pi7);
  373. tn8 = tan(8*pi7);
  374.  
  375.  
  376.  
  377.  
  378. }
  379.  
  380.  
  381.  
  382.  
  383.  
  384.  
  385. void comp_ram(int sel) {
  386.  
  387.  
  388. /*
  389. float pf, qf, rf, root, r1f, r2f;
  390. int p, q, rr, r2, r12, r22;
  391.  
  392. printf("alpha = %f\n", alpha);
  393. printf("beta = %f\n", beta);
  394. printf("gammma = %f\n", gamma);
  395.  
  396.  
  397.  
  398.  
  399. printf("***** (%d,%d) *****\n", mm, nn);
  400.  
  401.  
  402.  
  403. printf("alpha+beta+gamma = %f\n", alpha +beta+gamma);
  404. printf("alpha*beta+beta*gamma+gamma*alpha = %f\n", alpha *beta+beta*gamma+gamma*alpha );
  405. printf("alpha*beta*gamma = %f\n", alpha *beta*gamma);
  406.  
  407.  
  408. printf("alpha^(1/3) + beta^(1/3) + gamma^(1/3) = %f\n",
  409. croot(alpha) + croot(beta) + croot(gamma));
  410.  
  411. printf("(1/alpha)^(1/3) + (1/beta)^(1/3) + 1/(gamma)^(1/3) = %f\n",
  412. croot(1.0/alpha) + croot(1.0/beta) + croot(1.0/gamma));
  413.  
  414. */
  415.  
  416. a = cvt(alpha +beta+gamma);
  417. b = cvt(alpha *beta+beta*gamma+gamma*alpha);
  418.  
  419.  
  420.  
  421.  
  422.  
  423. /*
  424.  
  425. // printf("a = %d, b = %d\n", a, b);
  426.  
  427. // x = k*k*k - 3*(a+b+3)*k - (a*b + 6*(a + b) + 9);
  428.  
  429. printf("Ramanujan eq: x^3 - %d x - %d = 0 \n",
  430.  3*(a+b+3), a*b + 6*(a + b) + 9);
  431.  
  432. // x^3 + px = q = 0
  433. // Cardano formula
  434. // root = (-q/2 + (q^2/4+p^3/27)^(1/2))^(1/3) + (-q/2 - (q^2/4+p^3/27)^(1/2))^(1/3)
  435. // r = (q^2/4+p^3/27)^(1/2)
  436. p = -(3*(a+b+3));
  437. q = -(a*b + 6*(a + b) + 9);
  438.  
  439. printf("p = %d, q = %d\n", p, q);
  440. pf = 1.0*p;
  441. qf = 1.0*q;
  442.  
  443. rf = sqrt( pwrf(qf,2)/4.0 + pwrf(pf,3)/27.0 );
  444. r2 = q*q + 4*p*p*p /27;
  445. rr = cvt(sqrt(1.0*r2));
  446.  
  447. printf(" rr = sqrt(%d)/2 = %d/2\n", r2, rr );
  448. printf(" rf = %f\n", rf);
  449.  
  450. r12 = (-q + rr)/2;
  451. r22 = (-q - rr)/2;
  452.  
  453. printf("r12 = %d\n", r12);
  454. printf("r22 = %d\n", r22);
  455.  
  456.  
  457. r1f = croot(-qf/2.0 + rf );
  458. r2f = croot(-qf/2.0 - rf );
  459.  
  460. printf(" r12 = (%d)^(1/3) \n", r12);
  461. printf(" r22 = (%d)^(1/3) \n", r22);
  462. printf("root = (%d)^(1/3) + (%d)^(1/3)\n", r12, r22);
  463.  
  464. root = r1f + r2f;
  465. printf("root = %f, %f + %f \n", root, r1f, r2f);
  466.  
  467. */
  468.  
  469.  
  470. process(sel);
  471.  
  472.  
  473. }
  474.  
  475. void set_sin_mn(int m, int n) {
  476.  
  477.  
  478. alpha = pwrf(sn2, m)/(pwrf(sn4, n))*(pwrf(sn8, n-m));
  479. beta = pwrf(sn4, m)/(pwrf(sn8, n))*(pwrf(sn2, n-m));
  480. gamma = pwrf(sn8, m)/(pwrf(sn2, n))*(pwrf(sn4, n-m));
  481.  
  482.  
  483. }
  484.  
  485.  
  486.  
  487. ///////////////////////////////////////////////
  488.  
  489.  
  490.  
  491. int main(int argc, char *argv[]) {
  492. int i, j;
  493. int nterm = 100;
  494. init();
  495.  
  496.  
  497. for (i=0; i<nterm; i++) {
  498. for (j=0; j<nterm; j++) {
  499. mm = i;
  500. nn = j;
  501.  
  502. set_sin_mn(i, j);
  503. comp_ram(2);
  504.  
  505. }
  506. }
  507.  
  508.  
  509.  
  510.  
  511. printf("End of JOB (%d,%d)\n", mm, nn);
  512. return 1;
  513.  
  514. }
  515.  
  516.  
Runtime error #stdin #stdout 0s 9432KB
stdin
Standard input is empty
stdout
\begin{theorem} \label{tmn_0_0} 
Let 
$$
\alpha = \frac{(2\cos(2\theta)^0}{(2\cos(4\theta)^0},
\beta  = \frac{(2\cos(4\theta)^0}{(2\cos(8\theta)^0},
\gamma = \frac{(2\cos(8\theta)^0}{(2\cos(2\theta)^0}.
$$
Then $ \{ \alpha, \beta, \gamma \} $ are roots of the equation:
\begin{equation} \label{emn_0_0} 
x^3 - 3 x^2 + 3 x^2 - 1 = 0
\end{equation}  
The associatd Rammnujan equation has integer solution $ -3 $. 
And 
\begin{equation} \label{fmn_0_0} 
\sqrt[3]{\alpha} + \sqrt[3]{\beta} + \sqrt[3]{\gamma} = 
\sqrt[3]{0} 
\end{equation}  
\begin{equation} \label{gmn_0_0} 
\frac{1}{\sqrt[3]{\alpha}} + \frac{1}{\sqrt[3]{\beta}} + \frac{1}{\sqrt[3]{\gamma}} = 
 \sqrt[3]{0} 
\end{equation}  
\end{theorem}

\begin{theorem} \label{tmn_0_3} 
Let 
$$
\alpha = \frac{(2\cos(2\theta)^0}{(2\cos(4\theta)^3},
\beta  = \frac{(2\cos(4\theta)^0}{(2\cos(8\theta)^3},
\gamma = \frac{(2\cos(8\theta)^0}{(2\cos(2\theta)^3}.
$$
Then $ \{ \alpha, \beta, \gamma \} $ are roots of the equation:
\begin{equation} \label{emn_0_3} 
x^3 + 4 x^2 - 11 x - 1 = 0
\end{equation}  
The associatd Rammnujan equation has integer solution $ -1 $. 
And 
\begin{equation} \label{fmn_0_3} 
\sqrt[3]{\alpha} + \sqrt[3]{\beta} + \sqrt[3]{\gamma} = 
-\sqrt[3]{1} 
\end{equation}  
\begin{equation} \label{gmn_0_3} 
\frac{1}{\sqrt[3]{\alpha}} + \frac{1}{\sqrt[3]{\beta}} + \frac{1}{\sqrt[3]{\gamma}} = 
-\sqrt[3]{8} 
\end{equation}  
\end{theorem}

\begin{theorem} \label{tmn_0_6} 
Let 
$$
\alpha = \frac{(2\cos(2\theta)^0}{(2\cos(4\theta)^6},
\beta  = \frac{(2\cos(4\theta)^0}{(2\cos(8\theta)^6},
\gamma = \frac{(2\cos(8\theta)^0}{(2\cos(2\theta)^6}.
$$
Then $ \{ \alpha, \beta, \gamma \} $ are roots of the equation:
\begin{equation} \label{emn_0_6} 
x^3 - 38 x^2 + 129 x^2 - 1 = 0
\end{equation}  
The associatd Rammnujan equation has integer solution $ 27 $. 
And 
\begin{equation} \label{fmn_0_6} 
\sqrt[3]{\alpha} + \sqrt[3]{\beta} + \sqrt[3]{\gamma} = 
\sqrt[3]{125} 
\end{equation}  
\begin{equation} \label{gmn_0_6} 
\frac{1}{\sqrt[3]{\alpha}} + \frac{1}{\sqrt[3]{\beta}} + \frac{1}{\sqrt[3]{\gamma}} = 
 \sqrt[3]{216} 
\end{equation}  
\end{theorem}

\begin{theorem} \label{tmn_0_9} 
Let 
$$
\alpha = \frac{(2\cos(2\theta)^0}{(2\cos(4\theta)^9},
\beta  = \frac{(2\cos(4\theta)^0}{(2\cos(8\theta)^9},
\gamma = \frac{(2\cos(8\theta)^0}{(2\cos(2\theta)^9}.
$$
Then $ \{ \alpha, \beta, \gamma \} $ are roots of the equation:
\begin{equation} \label{emn_0_9} 
x^3 + 193 x^2 - 1460 x - 1 = 0
\end{equation}  
The associatd Rammnujan equation has integer solution $ 41 $. 
And 
\begin{equation} \label{fmn_0_9} 
\sqrt[3]{\alpha} + \sqrt[3]{\beta} + \sqrt[3]{\gamma} = 
-\sqrt[3]{64} 
\end{equation}  
\begin{equation} \label{gmn_0_9} 
\frac{1}{\sqrt[3]{\alpha}} + \frac{1}{\sqrt[3]{\beta}} + \frac{1}{\sqrt[3]{\gamma}} = 
-\sqrt[3]{1331} 
\end{equation}  
\end{theorem}

\begin{theorem} \label{tmn_0_12} 
Let 
$$
\alpha = \frac{(2\cos(2\theta)^0}{(2\cos(4\theta)^12},
\beta  = \frac{(2\cos(4\theta)^0}{(2\cos(8\theta)^12},
\gamma = \frac{(2\cos(8\theta)^0}{(2\cos(2\theta)^12}.
$$
Then $ \{ \alpha, \beta, \gamma \} $ are roots of the equation:
\begin{equation} \label{emn_0_12} 
x^3 - 1186 x^2 + 16565 x^2 - 1 = 0
\end{equation}  
The associatd Rammnujan equation has integer solution $ 335 $. 
And 
\begin{equation} \label{fmn_0_12} 
\sqrt[3]{\alpha} + \sqrt[3]{\beta} + \sqrt[3]{\gamma} = 
\sqrt[3]{2197} 
\end{equation}  
\begin{equation} \label{gmn_0_12} 
\frac{1}{\sqrt[3]{\alpha}} + \frac{1}{\sqrt[3]{\beta}} + \frac{1}{\sqrt[3]{\gamma}} = 
 \sqrt[3]{17576} 
\end{equation}  
\end{theorem}

\begin{theorem} \label{tmn_0_15} 
Let 
$$
\alpha = \frac{(2\cos(2\theta)^0}{(2\cos(4\theta)^15},
\beta  = \frac{(2\cos(4\theta)^0}{(2\cos(8\theta)^15},
\gamma = \frac{(2\cos(8\theta)^0}{(2\cos(2\theta)^15}.
$$
Then $ \{ \alpha, \beta, \gamma \} $ are roots of the equation:
\begin{equation} \label{emn_0_15} 
x^3 + 6829 x^2 - 187926 x - 1 = 0
\end{equation}  
The associatd Rammnujan equation has integer solution $ 909 $. 
And 
\begin{equation} \label{fmn_0_15} 
\sqrt[3]{\alpha} + \sqrt[3]{\beta} + \sqrt[3]{\gamma} = 
-\sqrt[3]{4096} 
\end{equation}  
\begin{equation} \label{gmn_0_15} 
\frac{1}{\sqrt[3]{\alpha}} + \frac{1}{\sqrt[3]{\beta}} + \frac{1}{\sqrt[3]{\gamma}} = 
-\sqrt[3]{185193} 
\end{equation}  
\end{theorem}

\begin{theorem} \label{tmn_1_2} 
Let 
$$
\alpha = \frac{(2\cos(2\theta)^1}{(2\cos(4\theta)^2},
\beta  = \frac{(2\cos(4\theta)^1}{(2\cos(8\theta)^2},
\gamma = \frac{(2\cos(8\theta)^1}{(2\cos(2\theta)^2}.
$$
Then $ \{ \alpha, \beta, \gamma \} $ are roots of the equation:
\begin{equation} \label{emn_1_2} 
x^3 - 3 x^2 - 4 x - 1 = 0
\end{equation}  
The associatd Rammnujan equation has integer solution $ -3 $. 
And 
\begin{equation} \label{fmn_1_2} 
\sqrt[3]{\alpha} + \sqrt[3]{\beta} + \sqrt[3]{\gamma} = 
\sqrt[3]{0} 
\end{equation}  
\begin{equation} \label{gmn_1_2} 
\frac{1}{\sqrt[3]{\alpha}} + \frac{1}{\sqrt[3]{\beta}} + \frac{1}{\sqrt[3]{\gamma}} = 
-\sqrt[3]{7} 
\end{equation}  
\end{theorem}

\begin{theorem} \label{tmn_1_5} 
Let 
$$
\alpha = \frac{(2\cos(2\theta)^1}{(2\cos(4\theta)^5},
\beta  = \frac{(2\cos(4\theta)^1}{(2\cos(8\theta)^5},
\gamma = \frac{(2\cos(8\theta)^1}{(2\cos(2\theta)^5}.
$$
Then $ \{ \alpha, \beta, \gamma \} $ are roots of the equation:
\begin{equation} \label{emn_1_5} 
x^3 + 25 x^2 + 31 x^2 - 1 = 0
\end{equation}  
The associatd Rammnujan equation has integer solution $ -10 $. 
And 
\begin{equation} \label{fmn_1_5} 
\sqrt[3]{\alpha} + \sqrt[3]{\beta} + \sqrt[3]{\gamma} = 
-\sqrt[3]{49} 
\end{equation}  
\begin{equation} \label{gmn_1_5} 
\frac{1}{\sqrt[3]{\alpha}} + \frac{1}{\sqrt[3]{\beta}} + \frac{1}{\sqrt[3]{\gamma}} = 
 \sqrt[3]{7} 
\end{equation}  
\end{theorem}

\begin{theorem} \label{tmn_1_8} 
Let 
$$
\alpha = \frac{(2\cos(2\theta)^1}{(2\cos(4\theta)^8},
\beta  = \frac{(2\cos(4\theta)^1}{(2\cos(8\theta)^8},
\gamma = \frac{(2\cos(8\theta)^1}{(2\cos(2\theta)^8}.
$$
Then $ \{ \alpha, \beta, \gamma \} $ are roots of the equation:
\begin{equation} \label{emn_1_8} 
x^3 - 136 x^2 - 361 x - 1 = 0
\end{equation}  
The associatd Rammnujan equation has integer solution $ -31 $. 
And 
\begin{equation} \label{fmn_1_8} 
\sqrt[3]{\alpha} + \sqrt[3]{\beta} + \sqrt[3]{\gamma} = 
\sqrt[3]{49} 
\end{equation}  
\begin{equation} \label{gmn_1_8} 
\frac{1}{\sqrt[3]{\alpha}} + \frac{1}{\sqrt[3]{\beta}} + \frac{1}{\sqrt[3]{\gamma}} = 
-\sqrt[3]{448} 
\end{equation}  
\end{theorem}

\begin{theorem} \label{tmn_1_11} 
Let 
$$
\alpha = \frac{(2\cos(2\theta)^1}{(2\cos(4\theta)^11},
\beta  = \frac{(2\cos(4\theta)^1}{(2\cos(8\theta)^11},
\gamma = \frac{(2\cos(8\theta)^1}{(2\cos(2\theta)^11}.
$$
Then $ \{ \alpha, \beta, \gamma \} $ are roots of the equation:
\begin{equation} \label{emn_1_11} 
x^3 + 816 x^2 + 4091 x^2 - 1 = 0
\end{equation}  
The associatd Rammnujan equation has integer solution $ -171 $. 
And 
\begin{equation} \label{fmn_1_11} 
\sqrt[3]{\alpha} + \sqrt[3]{\beta} + \sqrt[3]{\gamma} = 
-\sqrt[3]{1323} 
\end{equation}  
\begin{equation} \label{gmn_1_11} 
\frac{1}{\sqrt[3]{\alpha}} + \frac{1}{\sqrt[3]{\beta}} + \frac{1}{\sqrt[3]{\gamma}} = 
 \sqrt[3]{3584} 
\end{equation}  
\end{theorem}

\begin{theorem} \label{tmn_1_14} 
Let 
$$
\alpha = \frac{(2\cos(2\theta)^1}{(2\cos(4\theta)^14},
\beta  = \frac{(2\cos(4\theta)^1}{(2\cos(8\theta)^14},
\gamma = \frac{(2\cos(8\theta)^1}{(2\cos(2\theta)^14}.
$$
Then $ \{ \alpha, \beta, \gamma \} $ are roots of the equation:
\begin{equation} \label{emn_1_14} 
x^3 - 4735 x^2 - 46414 x - 1 = 0
\end{equation}  
The associatd Rammnujan equation has integer solution $ -535 $. 
And 
\begin{equation} \label{fmn_1_14} 
\sqrt[3]{\alpha} + \sqrt[3]{\beta} + \sqrt[3]{\gamma} = 
\sqrt[3]{3136} 
\end{equation}  
\begin{equation} \label{gmn_1_14} 
\frac{1}{\sqrt[3]{\alpha}} + \frac{1}{\sqrt[3]{\beta}} + \frac{1}{\sqrt[3]{\gamma}} = 
-\sqrt[3]{48013} 
\end{equation}  
\end{theorem}

\begin{theorem} \label{tmn_2_1} 
Let 
$$
\alpha = \frac{(2\cos(2\theta)^2}{(2\cos(4\theta)^1},
\beta  = \frac{(2\cos(4\theta)^2}{(2\cos(8\theta)^1},
\gamma = \frac{(2\cos(8\theta)^2}{(2\cos(2\theta)^1}.
$$
Then $ \{ \alpha, \beta, \gamma \} $ are roots of the equation:
\begin{equation} \label{emn_2_1} 
x^3 + 4 x^2 + 3 x^2 - 1 = 0
\end{equation}  
The associatd Rammnujan equation has integer solution $ -3 $. 
And 
\begin{equation} \label{fmn_2_1} 
\sqrt[3]{\alpha} + \sqrt[3]{\beta} + \sqrt[3]{\gamma} = 
-\sqrt[3]{7} 
\end{equation}  
\begin{equation} \label{gmn_2_1} 
\frac{1}{\sqrt[3]{\alpha}} + \frac{1}{\sqrt[3]{\beta}} + \frac{1}{\sqrt[3]{\gamma}} = 
 \sqrt[3]{0} 
\end{equation}  
\end{theorem}

\begin{theorem} \label{tmn_2_4} 
Let 
$$
\alpha = \frac{(2\cos(2\theta)^2}{(2\cos(4\theta)^4},
\beta  = \frac{(2\cos(4\theta)^2}{(2\cos(8\theta)^4},
\gamma = \frac{(2\cos(8\theta)^2}{(2\cos(2\theta)^4}.
$$
Then $ \{ \alpha, \beta, \gamma \} $ are roots of the equation:
\begin{equation} \label{emn_2_4} 
x^3 - 17 x^2 + 10 x^2 - 1 = 0
\end{equation}  
The associatd Rammnujan equation has integer solution $ 11 $. 
And 
\begin{equation} \label{fmn_2_4} 
\sqrt[3]{\alpha} + \sqrt[3]{\beta} + \sqrt[3]{\gamma} = 
\sqrt[3]{56} 
\end{equation}  
\begin{equation} \label{gmn_2_4} 
\frac{1}{\sqrt[3]{\alpha}} + \frac{1}{\sqrt[3]{\beta}} + \frac{1}{\sqrt[3]{\gamma}} = 
 \sqrt[3]{49} 
\end{equation}  
\end{theorem}

\begin{theorem} \label{tmn_2_7} 
Let 
$$
\alpha = \frac{(2\cos(2\theta)^2}{(2\cos(4\theta)^7},
\beta  = \frac{(2\cos(4\theta)^2}{(2\cos(8\theta)^7},
\gamma = \frac{(2\cos(8\theta)^2}{(2\cos(2\theta)^7}.
$$
Then $ \{ \alpha, \beta, \gamma \} $ are roots of the equation:
\begin{equation} \label{emn_2_7} 
x^3 + 95 x^2 - 88 x - 1 = 0
\end{equation}  
The associatd Rammnujan equation has integer solution $ 11 $. 
And 
\begin{equation} \label{fmn_2_7} 
\sqrt[3]{\alpha} + \sqrt[3]{\beta} + \sqrt[3]{\gamma} = 
-\sqrt[3]{56} 
\end{equation}  
\begin{equation} \label{gmn_2_7} 
\frac{1}{\sqrt[3]{\alpha}} + \frac{1}{\sqrt[3]{\beta}} + \frac{1}{\sqrt[3]{\gamma}} = 
-\sqrt[3]{49} 
\end{equation}  
\end{theorem}

\begin{theorem} \label{tmn_2_10} 
Let 
$$
\alpha = \frac{(2\cos(2\theta)^2}{(2\cos(4\theta)^10},
\beta  = \frac{(2\cos(4\theta)^2}{(2\cos(8\theta)^10},
\gamma = \frac{(2\cos(8\theta)^2}{(2\cos(2\theta)^10}.
$$
Then $ \{ \alpha, \beta, \gamma \} $ are roots of the equation:
\begin{equation} \label{emn_2_10} 
x^3 - 563 x^2 + 1011 x^2 - 1 = 0
\end{equation}  
The associatd Rammnujan equation has integer solution $ 102 $. 
And 
\begin{equation} \label{fmn_2_10} 
\sqrt[3]{\alpha} + \sqrt[3]{\beta} + \sqrt[3]{\gamma} = 
\sqrt[3]{875} 
\end{equation}  
\begin{equation} \label{gmn_2_10} 
\frac{1}{\sqrt[3]{\alpha}} + \frac{1}{\sqrt[3]{\beta}} + \frac{1}{\sqrt[3]{\gamma}} = 
 \sqrt[3]{1323} 
\end{equation}  
\end{theorem}

\begin{theorem} \label{tmn_2_13} 
Let 
$$
\alpha = \frac{(2\cos(2\theta)^2}{(2\cos(4\theta)^13},
\beta  = \frac{(2\cos(4\theta)^2}{(2\cos(8\theta)^13},
\gamma = \frac{(2\cos(8\theta)^2}{(2\cos(2\theta)^13}.
$$
Then $ \{ \alpha, \beta, \gamma \} $ are roots of the equation:
\begin{equation} \label{emn_2_13} 
x^3 + 3280 x^2 - 11463 x - 1 = 0
\end{equation}  
The associatd Rammnujan equation has integer solution $ 291 $. 
And 
\begin{equation} \label{fmn_2_13} 
\sqrt[3]{\alpha} + \sqrt[3]{\beta} + \sqrt[3]{\gamma} = 
-\sqrt[3]{2401} 
\end{equation}  
\begin{equation} \label{gmn_2_13} 
\frac{1}{\sqrt[3]{\alpha}} + \frac{1}{\sqrt[3]{\beta}} + \frac{1}{\sqrt[3]{\gamma}} = 
-\sqrt[3]{10584} 
\end{equation}  
\end{theorem}

\begin{theorem} \label{tmn_3_0} 
Let 
$$
\alpha = \frac{(2\cos(2\theta)^3}{(2\cos(4\theta)^0},
\beta  = \frac{(2\cos(4\theta)^3}{(2\cos(8\theta)^0},
\gamma = \frac{(2\cos(8\theta)^3}{(2\cos(2\theta)^0}.
$$
Then $ \{ \alpha, \beta, \gamma \} $ are roots of the equation:
\begin{equation} \label{emn_3_0} 
x^3 + 4 x^2 - 11 x - 1 = 0
\end{equation}  
The associatd Rammnujan equation has integer solution $ -1 $. 
And 
\begin{equation} \label{fmn_3_0} 
\sqrt[3]{\alpha} + \sqrt[3]{\beta} + \sqrt[3]{\gamma} = 
-\sqrt[3]{1} 
\end{equation}  
\begin{equation} \label{gmn_3_0} 
\frac{1}{\sqrt[3]{\alpha}} + \frac{1}{\sqrt[3]{\beta}} + \frac{1}{\sqrt[3]{\gamma}} = 
-\sqrt[3]{8} 
\end{equation}  
\end{theorem}

\begin{theorem} \label{tmn_3_3} 
Let 
$$
\alpha = \frac{(2\cos(2\theta)^3}{(2\cos(4\theta)^3},
\beta  = \frac{(2\cos(4\theta)^3}{(2\cos(8\theta)^3},
\gamma = \frac{(2\cos(8\theta)^3}{(2\cos(2\theta)^3}.
$$
Then $ \{ \alpha, \beta, \gamma \} $ are roots of the equation:
\begin{equation} \label{emn_3_3} 
x^3 + 11 x^2 - 4 x - 1 = 0
\end{equation}  
The associatd Rammnujan equation has integer solution $ -1 $. 
And 
\begin{equation} \label{fmn_3_3} 
\sqrt[3]{\alpha} + \sqrt[3]{\beta} + \sqrt[3]{\gamma} = 
-\sqrt[3]{8} 
\end{equation}  
\begin{equation} \label{gmn_3_3} 
\frac{1}{\sqrt[3]{\alpha}} + \frac{1}{\sqrt[3]{\beta}} + \frac{1}{\sqrt[3]{\gamma}} = 
-\sqrt[3]{1} 
\end{equation}  
\end{theorem}

\begin{theorem} \label{tmn_3_6} 
Let 
$$
\alpha = \frac{(2\cos(2\theta)^3}{(2\cos(4\theta)^6},
\beta  = \frac{(2\cos(4\theta)^3}{(2\cos(8\theta)^6},
\gamma = \frac{(2\cos(8\theta)^3}{(2\cos(2\theta)^6}.
$$
Then $ \{ \alpha, \beta, \gamma \} $ are roots of the equation:
\begin{equation} \label{emn_3_6} 
x^3 - 66 x^2 - 25 x - 1 = 0
\end{equation}  
The associatd Rammnujan equation has integer solution $ -15 $. 
And 
\begin{equation} \label{fmn_3_6} 
\sqrt[3]{\alpha} + \sqrt[3]{\beta} + \sqrt[3]{\gamma} = 
\sqrt[3]{27} 
\end{equation}  
\begin{equation} \label{gmn_3_6} 
\frac{1}{\sqrt[3]{\alpha}} + \frac{1}{\sqrt[3]{\beta}} + \frac{1}{\sqrt[3]{\gamma}} = 
-\sqrt[3]{64} 
\end{equation}  
\end{theorem}

\begin{theorem} \label{tmn_3_9} 
Let 
$$
\alpha = \frac{(2\cos(2\theta)^3}{(2\cos(4\theta)^9},
\beta  = \frac{(2\cos(4\theta)^3}{(2\cos(8\theta)^9},
\gamma = \frac{(2\cos(8\theta)^3}{(2\cos(2\theta)^9}.
$$
Then $ \{ \alpha, \beta, \gamma \} $ are roots of the equation:
\begin{equation} \label{emn_3_9} 
x^3 + 389 x^2 + 248 x^2 - 1 = 0
\end{equation}  
The associatd Rammnujan equation has integer solution $ -43 $. 
And 
\begin{equation} \label{fmn_3_9} 
\sqrt[3]{\alpha} + \sqrt[3]{\beta} + \sqrt[3]{\gamma} = 
-\sqrt[3]{512} 
\end{equation}  
\begin{equation} \label{gmn_3_9} 
\frac{1}{\sqrt[3]{\alpha}} + \frac{1}{\sqrt[3]{\beta}} + \frac{1}{\sqrt[3]{\gamma}} = 
 \sqrt[3]{125} 
\end{equation}  
\end{theorem}

\begin{theorem} \label{tmn_3_12} 
Let 
$$
\alpha = \frac{(2\cos(2\theta)^3}{(2\cos(4\theta)^12},
\beta  = \frac{(2\cos(4\theta)^3}{(2\cos(8\theta)^12},
\gamma = \frac{(2\cos(8\theta)^3}{(2\cos(2\theta)^12}.
$$
Then $ \{ \alpha, \beta, \gamma \} $ are roots of the equation:
\begin{equation} \label{emn_3_12} 
x^3 - 2271 x^2 - 2832 x - 1 = 0
\end{equation}  
The associatd Rammnujan equation has integer solution $ -183 $. 
And 
\begin{equation} \label{fmn_3_12} 
\sqrt[3]{\alpha} + \sqrt[3]{\beta} + \sqrt[3]{\gamma} = 
\sqrt[3]{1728} 
\end{equation}  
\begin{equation} \label{gmn_3_12} 
\frac{1}{\sqrt[3]{\alpha}} + \frac{1}{\sqrt[3]{\beta}} + \frac{1}{\sqrt[3]{\gamma}} = 
-\sqrt[3]{3375} 
\end{equation}  
\end{theorem}

\begin{theorem} \label{tmn_3_15} 
Let 
$$
\alpha = \frac{(2\cos(2\theta)^3}{(2\cos(4\theta)^15},
\beta  = \frac{(2\cos(4\theta)^3}{(2\cos(8\theta)^15},
\gamma = \frac{(2\cos(8\theta)^3}{(2\cos(2\theta)^15}.
$$
Then $ \{ \alpha, \beta, \gamma \} $ are roots of the equation:
\begin{equation} \label{emn_3_15} 
x^3 + 13297 x^2 + 32119 x^2 - 1 = 0
\end{equation}  
The associatd Rammnujan equation has integer solution $ -778 $. 
And 
\begin{equation} \label{fmn_3_15} 
\sqrt[3]{\alpha} + \sqrt[3]{\beta} + \sqrt[3]{\gamma} = 
-\sqrt[3]{15625} 
\end{equation}  
\begin{equation} \label{gmn_3_15} 
\frac{1}{\sqrt[3]{\alpha}} + \frac{1}{\sqrt[3]{\beta}} + \frac{1}{\sqrt[3]{\gamma}} = 
 \sqrt[3]{29791} 
\end{equation}  
\end{theorem}

\begin{theorem} \label{tmn_4_2} 
Let 
$$
\alpha = \frac{(2\cos(2\theta)^4}{(2\cos(4\theta)^2},
\beta  = \frac{(2\cos(4\theta)^4}{(2\cos(8\theta)^2},
\gamma = \frac{(2\cos(8\theta)^4}{(2\cos(2\theta)^2}.
$$
Then $ \{ \alpha, \beta, \gamma \} $ are roots of the equation:
\begin{equation} \label{emn_4_2} 
x^3 - 10 x^2 + 17 x^2 - 1 = 0
\end{equation}  
The associatd Rammnujan equation has integer solution $ 11 $. 
And 
\begin{equation} \label{fmn_4_2} 
\sqrt[3]{\alpha} + \sqrt[3]{\beta} + \sqrt[3]{\gamma} = 
\sqrt[3]{49} 
\end{equation}  
\begin{equation} \label{gmn_4_2} 
\frac{1}{\sqrt[3]{\alpha}} + \frac{1}{\sqrt[3]{\beta}} + \frac{1}{\sqrt[3]{\gamma}} = 
 \sqrt[3]{56} 
\end{equation}  
\end{theorem}

\begin{theorem} \label{tmn_4_5} 
Let 
$$
\alpha = \frac{(2\cos(2\theta)^4}{(2\cos(4\theta)^5},
\beta  = \frac{(2\cos(4\theta)^4}{(2\cos(8\theta)^5},
\gamma = \frac{(2\cos(8\theta)^4}{(2\cos(2\theta)^5}.
$$
Then $ \{ \alpha, \beta, \gamma \} $ are roots of the equation:
\begin{equation} \label{emn_4_5} 
x^3 + 46 x^2 + 3 x^2 - 1 = 0
\end{equation}  
The associatd Rammnujan equation has integer solution $ -3 $. 
And 
\begin{equation} \label{fmn_4_5} 
\sqrt[3]{\alpha} + \sqrt[3]{\beta} + \sqrt[3]{\gamma} = 
-\sqrt[3]{49} 
\end{equation}  
\begin{equation} \label{gmn_4_5} 
\frac{1}{\sqrt[3]{\alpha}} + \frac{1}{\sqrt[3]{\beta}} + \frac{1}{\sqrt[3]{\gamma}} = 
 \sqrt[3]{0} 
\end{equation}  
\end{theorem}

\begin{theorem} \label{tmn_4_8} 
Let 
$$
\alpha = \frac{(2\cos(2\theta)^4}{(2\cos(4\theta)^8},
\beta  = \frac{(2\cos(4\theta)^4}{(2\cos(8\theta)^8},
\gamma = \frac{(2\cos(8\theta)^4}{(2\cos(2\theta)^8}.
$$
Then $ \{ \alpha, \beta, \gamma \} $ are roots of the equation:
\begin{equation} \label{emn_4_8} 
x^3 - 269 x^2 + 66 x^2 - 1 = 0
\end{equation}  
The associatd Rammnujan equation has integer solution $ 39 $. 
And 
\begin{equation} \label{fmn_4_8} 
\sqrt[3]{\alpha} + \sqrt[3]{\beta} + \sqrt[3]{\gamma} = 
\sqrt[3]{392} 
\end{equation}  
\begin{equation} \label{gmn_4_8} 
\frac{1}{\sqrt[3]{\alpha}} + \frac{1}{\sqrt[3]{\beta}} + \frac{1}{\sqrt[3]{\gamma}} = 
 \sqrt[3]{189} 
\end{equation}  
\end{theorem}

\begin{theorem} \label{tmn_4_11} 
Let 
$$
\alpha = \frac{(2\cos(2\theta)^4}{(2\cos(4\theta)^11},
\beta  = \frac{(2\cos(4\theta)^4}{(2\cos(8\theta)^11},
\gamma = \frac{(2\cos(8\theta)^4}{(2\cos(2\theta)^11}.
$$
Then $ \{ \alpha, \beta, \gamma \} $ are roots of the equation:
\begin{equation} \label{emn_4_11} 
x^3 + 1572 x^2 - 697 x - 1 = 0
\end{equation}  
The associatd Rammnujan equation has integer solution $ 81 $. 
And 
\begin{equation} \label{fmn_4_11} 
\sqrt[3]{\alpha} + \sqrt[3]{\beta} + \sqrt[3]{\gamma} = 
-\sqrt[3]{1323} 
\end{equation}  
\begin{equation} \label{gmn_4_11} 
\frac{1}{\sqrt[3]{\alpha}} + \frac{1}{\sqrt[3]{\beta}} + \frac{1}{\sqrt[3]{\gamma}} = 
-\sqrt[3]{448} 
\end{equation}  
\end{theorem}

\begin{theorem} \label{tmn_4_14} 
Let 
$$
\alpha = \frac{(2\cos(2\theta)^4}{(2\cos(4\theta)^14},
\beta  = \frac{(2\cos(4\theta)^4}{(2\cos(8\theta)^14},
\gamma = \frac{(2\cos(8\theta)^4}{(2\cos(2\theta)^14}.
$$
Then $ \{ \alpha, \beta, \gamma \} $ are roots of the equation:
\begin{equation} \label{emn_4_14} 
x^3 - 9201 x^2 + 7934 x^2 - 1 = 0
\end{equation}  
The associatd Rammnujan equation has integer solution $ 459 $. 
And 
\begin{equation} \label{fmn_4_14} 
\sqrt[3]{\alpha} + \sqrt[3]{\beta} + \sqrt[3]{\gamma} = 
\sqrt[3]{10584} 
\end{equation}  
\begin{equation} \label{gmn_4_14} 
\frac{1}{\sqrt[3]{\alpha}} + \frac{1}{\sqrt[3]{\beta}} + \frac{1}{\sqrt[3]{\gamma}} = 
 \sqrt[3]{9317} 
\end{equation}  
\end{theorem}

\begin{theorem} \label{tmn_5_1} 
Let 
$$
\alpha = \frac{(2\cos(2\theta)^5}{(2\cos(4\theta)^1},
\beta  = \frac{(2\cos(4\theta)^5}{(2\cos(8\theta)^1},
\gamma = \frac{(2\cos(8\theta)^5}{(2\cos(2\theta)^1}.
$$
Then $ \{ \alpha, \beta, \gamma \} $ are roots of the equation:
\begin{equation} \label{emn_5_1} 
x^3 - 3 x^2 - 46 x - 1 = 0
\end{equation}  
The associatd Rammnujan equation has integer solution $ -3 $. 
And 
\begin{equation} \label{fmn_5_1} 
\sqrt[3]{\alpha} + \sqrt[3]{\beta} + \sqrt[3]{\gamma} = 
\sqrt[3]{0} 
\end{equation}  
\begin{equation} \label{gmn_5_1} 
\frac{1}{\sqrt[3]{\alpha}} + \frac{1}{\sqrt[3]{\beta}} + \frac{1}{\sqrt[3]{\gamma}} = 
-\sqrt[3]{49} 
\end{equation}  
\end{theorem}

\begin{theorem} \label{tmn_5_4} 
Let 
$$
\alpha = \frac{(2\cos(2\theta)^5}{(2\cos(4\theta)^4},
\beta  = \frac{(2\cos(4\theta)^5}{(2\cos(8\theta)^4},
\gamma = \frac{(2\cos(8\theta)^5}{(2\cos(2\theta)^4}.
$$
Then $ \{ \alpha, \beta, \gamma \} $ are roots of the equation:
\begin{equation} \label{emn_5_4} 
x^3 - 31 x^2 - 25 x - 1 = 0
\end{equation}  
The associatd Rammnujan equation has integer solution $ -10 $. 
And 
\begin{equation} \label{fmn_5_4} 
\sqrt[3]{\alpha} + \sqrt[3]{\beta} + \sqrt[3]{\gamma} = 
\sqrt[3]{7} 
\end{equation}  
\begin{equation} \label{gmn_5_4} 
\frac{1}{\sqrt[3]{\alpha}} + \frac{1}{\sqrt[3]{\beta}} + \frac{1}{\sqrt[3]{\gamma}} = 
-\sqrt[3]{49} 
\end{equation}  
\end{theorem}

\begin{theorem} \label{tmn_5_7} 
Let 
$$
\alpha = \frac{(2\cos(2\theta)^5}{(2\cos(4\theta)^7},
\beta  = \frac{(2\cos(4\theta)^5}{(2\cos(8\theta)^7},
\gamma = \frac{(2\cos(8\theta)^5}{(2\cos(2\theta)^7}.
$$
Then $ \{ \alpha, \beta, \gamma \} $ are roots of the equation:
\begin{equation} \label{emn_5_7} 
x^3 + 186 x^2 + 3 x^2 - 1 = 0
\end{equation}  
The associatd Rammnujan equation has integer solution $ -3 $. 
And 
\begin{equation} \label{fmn_5_7} 
\sqrt[3]{\alpha} + \sqrt[3]{\beta} + \sqrt[3]{\gamma} = 
-\sqrt[3]{189} 
\end{equation}  
\begin{equation} \label{gmn_5_7} 
\frac{1}{\sqrt[3]{\alpha}} + \frac{1}{\sqrt[3]{\beta}} + \frac{1}{\sqrt[3]{\gamma}} = 
 \sqrt[3]{0} 
\end{equation}  
\end{theorem}

\begin{theorem} \label{tmn_5_10} 
Let 
$$
\alpha = \frac{(2\cos(2\theta)^5}{(2\cos(4\theta)^10},
\beta  = \frac{(2\cos(4\theta)^5}{(2\cos(8\theta)^10},
\gamma = \frac{(2\cos(8\theta)^5}{(2\cos(2\theta)^10}.
$$
Then $ \{ \alpha, \beta, \gamma \} $ are roots of the equation:
\begin{equation} \label{emn_5_10} 
x^3 - 1088 x^2 - 179 x - 1 = 0
\end{equation}  
The associatd Rammnujan equation has integer solution $ -73 $. 
And 
\begin{equation} \label{fmn_5_10} 
\sqrt[3]{\alpha} + \sqrt[3]{\beta} + \sqrt[3]{\gamma} = 
\sqrt[3]{875} 
\end{equation}  
\begin{equation} \label{gmn_5_10} 
\frac{1}{\sqrt[3]{\alpha}} + \frac{1}{\sqrt[3]{\beta}} + \frac{1}{\sqrt[3]{\gamma}} = 
-\sqrt[3]{392} 
\end{equation}  
\end{theorem}

\begin{theorem} \label{tmn_5_13} 
Let 
$$
\alpha = \frac{(2\cos(2\theta)^5}{(2\cos(4\theta)^13},
\beta  = \frac{(2\cos(4\theta)^5}{(2\cos(8\theta)^13},
\gamma = \frac{(2\cos(8\theta)^5}{(2\cos(2\theta)^13}.
$$
Then $ \{ \alpha, \beta, \gamma \} $ are roots of the equation:
\begin{equation} \label{emn_5_13} 
x^3 + 6367 x^2 + 1956 x^2 - 1 = 0
\end{equation}  
The associatd Rammnujan equation has integer solution $ -213 $. 
And 
\begin{equation} \label{fmn_5_13} 
\sqrt[3]{\alpha} + \sqrt[3]{\beta} + \sqrt[3]{\gamma} = 
-\sqrt[3]{7000} 
\end{equation}  
\begin{equation} \label{gmn_5_13} 
\frac{1}{\sqrt[3]{\alpha}} + \frac{1}{\sqrt[3]{\beta}} + \frac{1}{\sqrt[3]{\gamma}} = 
 \sqrt[3]{1323} 
\end{equation}  
\end{theorem}

\begin{theorem} \label{tmn_5_16} 
Let 
$$
\alpha = \frac{(2\cos(2\theta)^5}{(2\cos(4\theta)^16},
\beta  = \frac{(2\cos(4\theta)^5}{(2\cos(8\theta)^16},
\gamma = \frac{(2\cos(8\theta)^5}{(2\cos(2\theta)^16}.
$$
Then $ \{ \alpha, \beta, \gamma \} $ are roots of the equation:
\begin{equation} \label{emn_5_16} 
x^3 - 37250 x^2 - 22229 x - 1 = 0
\end{equation}  
The associatd Rammnujan equation has integer solution $ -955 $. 
And 
\begin{equation} \label{fmn_5_16} 
\sqrt[3]{\alpha} + \sqrt[3]{\beta} + \sqrt[3]{\gamma} = 
\sqrt[3]{34391} 
\end{equation}  
\begin{equation} \label{gmn_5_16} 
\frac{1}{\sqrt[3]{\alpha}} + \frac{1}{\sqrt[3]{\beta}} + \frac{1}{\sqrt[3]{\gamma}} = 
-\sqrt[3]{25088} 
\end{equation}  
\end{theorem}

\begin{theorem} \label{tmn_6_0} 
Let 
$$
\alpha = \frac{(2\cos(2\theta)^6}{(2\cos(4\theta)^0},
\beta  = \frac{(2\cos(4\theta)^6}{(2\cos(8\theta)^0},
\gamma = \frac{(2\cos(8\theta)^6}{(2\cos(2\theta)^0}.
$$
Then $ \{ \alpha, \beta, \gamma \} $ are roots of the equation:
\begin{equation} \label{emn_6_0} 
x^3 - 38 x^2 + 129 x^2 - 1 = 0
\end{equation}  
The associatd Rammnujan equation has integer solution $ 27 $. 
And 
\begin{equation} \label{fmn_6_0} 
\sqrt[3]{\alpha} + \sqrt[3]{\beta} + \sqrt[3]{\gamma} = 
\sqrt[3]{125} 
\end{equation}  
\begin{equation} \label{gmn_6_0} 
\frac{1}{\sqrt[3]{\alpha}} + \frac{1}{\sqrt[3]{\beta}} + \frac{1}{\sqrt[3]{\gamma}} = 
 \sqrt[3]{216} 
\end{equation}  
\end{theorem}

\begin{theorem} \label{tmn_6_3} 
Let 
$$
\alpha = \frac{(2\cos(2\theta)^6}{(2\cos(4\theta)^3},
\beta  = \frac{(2\cos(4\theta)^6}{(2\cos(8\theta)^3},
\gamma = \frac{(2\cos(8\theta)^6}{(2\cos(2\theta)^3}.
$$
Then $ \{ \alpha, \beta, \gamma \} $ are roots of the equation:
\begin{equation} \label{emn_6_3} 
x^3 + 25 x^2 + 66 x^2 - 1 = 0
\end{equation}  
The associatd Rammnujan equation has integer solution $ -15 $. 
And 
\begin{equation} \label{fmn_6_3} 
\sqrt[3]{\alpha} + \sqrt[3]{\beta} + \sqrt[3]{\gamma} = 
-\sqrt[3]{64} 
\end{equation}  
\begin{equation} \label{gmn_6_3} 
\frac{1}{\sqrt[3]{\alpha}} + \frac{1}{\sqrt[3]{\beta}} + \frac{1}{\sqrt[3]{\gamma}} = 
 \sqrt[3]{27} 
\end{equation}  
\end{theorem}

\begin{theorem} \label{tmn_6_6} 
Let 
$$
\alpha = \frac{(2\cos(2\theta)^6}{(2\cos(4\theta)^6},
\beta  = \frac{(2\cos(4\theta)^6}{(2\cos(8\theta)^6},
\gamma = \frac{(2\cos(8\theta)^6}{(2\cos(2\theta)^6}.
$$
Then $ \{ \alpha, \beta, \gamma \} $ are roots of the equation:
\begin{equation} \label{emn_6_6} 
x^3 - 129 x^2 + 38 x^2 - 1 = 0
\end{equation}  
The associatd Rammnujan equation has integer solution $ 27 $. 
And 
\begin{equation} \label{fmn_6_6} 
\sqrt[3]{\alpha} + \sqrt[3]{\beta} + \sqrt[3]{\gamma} = 
\sqrt[3]{216} 
\end{equation}  
\begin{equation} \label{gmn_6_6} 
\frac{1}{\sqrt[3]{\alpha}} + \frac{1}{\sqrt[3]{\beta}} + \frac{1}{\sqrt[3]{\gamma}} = 
 \sqrt[3]{125} 
\end{equation}  
\end{theorem}

\begin{theorem} \label{tmn_6_9} 
Let 
$$
\alpha = \frac{(2\cos(2\theta)^6}{(2\cos(4\theta)^9},
\beta  = \frac{(2\cos(4\theta)^6}{(2\cos(8\theta)^9},
\gamma = \frac{(2\cos(8\theta)^6}{(2\cos(2\theta)^9}.
$$
Then $ \{ \alpha, \beta, \gamma \} $ are roots of the equation:
\begin{equation} \label{emn_6_9} 
x^3 + 753 x^2 - 25 x - 1 = 0
\end{equation}  
The associatd Rammnujan equation has integer solution $ 6 $. 
And 
\begin{equation} \label{fmn_6_9} 
\sqrt[3]{\alpha} + \sqrt[3]{\beta} + \sqrt[3]{\gamma} = 
-\sqrt[3]{729} 
\end{equation}  
\begin{equation} \label{gmn_6_9} 
\frac{1}{\sqrt[3]{\alpha}} + \frac{1}{\sqrt[3]{\beta}} + \frac{1}{\sqrt[3]{\gamma}} = 
-\sqrt[3]{1} 
\end{equation}  
\end{theorem}

\begin{theorem} \label{tmn_6_12} 
Let 
$$
\alpha = \frac{(2\cos(2\theta)^6}{(2\cos(4\theta)^12},
\beta  = \frac{(2\cos(4\theta)^6}{(2\cos(8\theta)^12},
\gamma = \frac{(2\cos(8\theta)^6}{(2\cos(2\theta)^12}.
$$
Then $ \{ \alpha, \beta, \gamma \} $ are roots of the equation:
\begin{equation} \label{emn_6_12} 
x^3 - 4406 x^2 + 493 x^2 - 1 = 0
\end{equation}  
The associatd Rammnujan equation has integer solution $ 167 $. 
And 
\begin{equation} \label{fmn_6_12} 
\sqrt[3]{\alpha} + \sqrt[3]{\beta} + \sqrt[3]{\gamma} = 
\sqrt[3]{4913} 
\end{equation}  
\begin{equation} \label{gmn_6_12} 
\frac{1}{\sqrt[3]{\alpha}} + \frac{1}{\sqrt[3]{\beta}} + \frac{1}{\sqrt[3]{\gamma}} = 
 \sqrt[3]{1000} 
\end{equation}  
\end{theorem}

\begin{theorem} \label{tmn_6_15} 
Let 
$$
\alpha = \frac{(2\cos(2\theta)^6}{(2\cos(4\theta)^15},
\beta  = \frac{(2\cos(4\theta)^6}{(2\cos(8\theta)^15},
\gamma = \frac{(2\cos(8\theta)^6}{(2\cos(2\theta)^15}.
$$
Then $ \{ \alpha, \beta, \gamma \} $ are roots of the equation:
\begin{equation} \label{emn_6_15} 
x^3 + 25778 x^2 - 5485 x - 1 = 0
\end{equation}  
The associatd Rammnujan equation has integer solution $ 461 $. 
And 
\begin{equation} \label{fmn_6_15} 
\sqrt[3]{\alpha} + \sqrt[3]{\beta} + \sqrt[3]{\gamma} = 
-\sqrt[3]{24389} 
\end{equation}  
\begin{equation} \label{gmn_6_15} 
\frac{1}{\sqrt[3]{\alpha}} + \frac{1}{\sqrt[3]{\beta}} + \frac{1}{\sqrt[3]{\gamma}} = 
-\sqrt[3]{4096} 
\end{equation}  
\end{theorem}

\begin{theorem} \label{tmn_7_2} 
Let 
$$
\alpha = \frac{(2\cos(2\theta)^7}{(2\cos(4\theta)^2},
\beta  = \frac{(2\cos(4\theta)^7}{(2\cos(8\theta)^2},
\gamma = \frac{(2\cos(8\theta)^7}{(2\cos(2\theta)^2}.
$$
Then $ \{ \alpha, \beta, \gamma \} $ are roots of the equation:
\begin{equation} \label{emn_7_2} 
x^3 - 3 x^2 - 186 x - 1 = 0
\end{equation}  
The associatd Rammnujan equation has integer solution $ -3 $. 
And 
\begin{equation} \label{fmn_7_2} 
\sqrt[3]{\alpha} + \sqrt[3]{\beta} + \sqrt[3]{\gamma} = 
\sqrt[3]{0} 
\end{equation}  
\begin{equation} \label{gmn_7_2} 
\frac{1}{\sqrt[3]{\alpha}} + \frac{1}{\sqrt[3]{\beta}} + \frac{1}{\sqrt[3]{\gamma}} = 
-\sqrt[3]{189} 
\end{equation}  
\end{theorem}

\begin{theorem} \label{tmn_7_5} 
Let 
$$
\alpha = \frac{(2\cos(2\theta)^7}{(2\cos(4\theta)^5},
\beta  = \frac{(2\cos(4\theta)^7}{(2\cos(8\theta)^5},
\gamma = \frac{(2\cos(8\theta)^7}{(2\cos(2\theta)^5}.
$$
Then $ \{ \alpha, \beta, \gamma \} $ are roots of the equation:
\begin{equation} \label{emn_7_5} 
x^3 + 88 x^2 - 95 x - 1 = 0
\end{equation}  
The associatd Rammnujan equation has integer solution $ 11 $. 
And 
\begin{equation} \label{fmn_7_5} 
\sqrt[3]{\alpha} + \sqrt[3]{\beta} + \sqrt[3]{\gamma} = 
-\sqrt[3]{49} 
\end{equation}  
\begin{equation} \label{gmn_7_5} 
\frac{1}{\sqrt[3]{\alpha}} + \frac{1}{\sqrt[3]{\beta}} + \frac{1}{\sqrt[3]{\gamma}} = 
-\sqrt[3]{56} 
\end{equation}  
\end{theorem}

\begin{theorem} \label{tmn_7_8} 
Let 
$$
\alpha = \frac{(2\cos(2\theta)^7}{(2\cos(4\theta)^8},
\beta  = \frac{(2\cos(4\theta)^7}{(2\cos(8\theta)^8},
\gamma = \frac{(2\cos(8\theta)^7}{(2\cos(2\theta)^8}.
$$
Then $ \{ \alpha, \beta, \gamma \} $ are roots of the equation:
\begin{equation} \label{emn_7_8} 
x^3 - 521 x^2 - 60 x - 1 = 0
\end{equation}  
The associatd Rammnujan equation has integer solution $ -45 $. 
And 
\begin{equation} \label{fmn_7_8} 
\sqrt[3]{\alpha} + \sqrt[3]{\beta} + \sqrt[3]{\gamma} = 
\sqrt[3]{392} 
\end{equation}  
\begin{equation} \label{gmn_7_8} 
\frac{1}{\sqrt[3]{\alpha}} + \frac{1}{\sqrt[3]{\beta}} + \frac{1}{\sqrt[3]{\gamma}} = 
-\sqrt[3]{189} 
\end{equation}  
\end{theorem}

\begin{theorem} \label{tmn_7_11} 
Let 
$$
\alpha = \frac{(2\cos(2\theta)^7}{(2\cos(4\theta)^11},
\beta  = \frac{(2\cos(4\theta)^7}{(2\cos(8\theta)^11},
\gamma = \frac{(2\cos(8\theta)^7}{(2\cos(2\theta)^11}.
$$
Then $ \{ \alpha, \beta, \gamma \} $ are roots of the equation:
\begin{equation} \label{emn_7_11} 
x^3 + 3049 x^2 + 94 x^2 - 1 = 0
\end{equation}  
The associatd Rammnujan equation has integer solution $ -31 $. 
And 
\begin{equation} \label{fmn_7_11} 
\sqrt[3]{\alpha} + \sqrt[3]{\beta} + \sqrt[3]{\gamma} = 
-\sqrt[3]{3136} 
\end{equation}  
\begin{equation} \label{gmn_7_11} 
\frac{1}{\sqrt[3]{\alpha}} + \frac{1}{\sqrt[3]{\beta}} + \frac{1}{\sqrt[3]{\gamma}} = 
 \sqrt[3]{7} 
\end{equation}  
\end{theorem}

\begin{theorem} \label{tmn_7_14} 
Let 
$$
\alpha = \frac{(2\cos(2\theta)^7}{(2\cos(4\theta)^14},
\beta  = \frac{(2\cos(4\theta)^7}{(2\cos(8\theta)^14},
\gamma = \frac{(2\cos(8\theta)^7}{(2\cos(2\theta)^14}.
$$
Then $ \{ \alpha, \beta, \gamma \} $ are roots of the equation:
\begin{equation} \label{emn_7_14} 
x^3 - 17839 x^2 - 1369 x - 1 = 0
\end{equation}  
The associatd Rammnujan equation has integer solution $ -346 $. 
And 
\begin{equation} \label{fmn_7_14} 
\sqrt[3]{\alpha} + \sqrt[3]{\beta} + \sqrt[3]{\gamma} = 
\sqrt[3]{16807} 
\end{equation}  
\begin{equation} \label{gmn_7_14} 
\frac{1}{\sqrt[3]{\alpha}} + \frac{1}{\sqrt[3]{\beta}} + \frac{1}{\sqrt[3]{\gamma}} = 
-\sqrt[3]{2401} 
\end{equation}  
\end{theorem}

\begin{theorem} \label{tmn_8_1} 
Let 
$$
\alpha = \frac{(2\cos(2\theta)^8}{(2\cos(4\theta)^1},
\beta  = \frac{(2\cos(4\theta)^8}{(2\cos(8\theta)^1},
\gamma = \frac{(2\cos(8\theta)^8}{(2\cos(2\theta)^1}.
$$
Then $ \{ \alpha, \beta, \gamma \} $ are roots of the equation:
\begin{equation} \label{emn_8_1} 
x^3 + 60 x^2 + 521 x^2 - 1 = 0
\end{equation}  
The associatd Rammnujan equation has integer solution $ -45 $. 
And 
\begin{equation} \label{fmn_8_1} 
\sqrt[3]{\alpha} + \sqrt[3]{\beta} + \sqrt[3]{\gamma} = 
-\sqrt[3]{189} 
\end{equation}  
\begin{equation} \label{gmn_8_1} 
\frac{1}{\sqrt[3]{\alpha}} + \frac{1}{\sqrt[3]{\beta}} + \frac{1}{\sqrt[3]{\gamma}} = 
 \sqrt[3]{392} 
\end{equation}  
\end{theorem}

\begin{theorem} \label{tmn_8_4} 
Let 
$$
\alpha = \frac{(2\cos(2\theta)^8}{(2\cos(4\theta)^4},
\beta  = \frac{(2\cos(4\theta)^8}{(2\cos(8\theta)^4},
\gamma = \frac{(2\cos(8\theta)^8}{(2\cos(2\theta)^4}.
$$
Then $ \{ \alpha, \beta, \gamma \} $ are roots of the equation:
\begin{equation} \label{emn_8_4} 
x^3 - 66 x^2 + 269 x^2 - 1 = 0
\end{equation}  
The associatd Rammnujan equation has integer solution $ 39 $. 
And 
\begin{equation} \label{fmn_8_4} 
\sqrt[3]{\alpha} + \sqrt[3]{\beta} + \sqrt[3]{\gamma} = 
\sqrt[3]{189} 
\end{equation}  
\begin{equation} \label{gmn_8_4} 
\frac{1}{\sqrt[3]{\alpha}} + \frac{1}{\sqrt[3]{\beta}} + \frac{1}{\sqrt[3]{\gamma}} = 
 \sqrt[3]{392} 
\end{equation}  
\end{theorem}

\begin{theorem} \label{tmn_8_7} 
Let 
$$
\alpha = \frac{(2\cos(2\theta)^8}{(2\cos(4\theta)^7},
\beta  = \frac{(2\cos(4\theta)^8}{(2\cos(8\theta)^7},
\gamma = \frac{(2\cos(8\theta)^8}{(2\cos(2\theta)^7}.
$$
Then $ \{ \alpha, \beta, \gamma \} $ are roots of the equation:
\begin{equation} \label{emn_8_7} 
x^3 + 361 x^2 + 136 x^2 - 1 = 0
\end{equation}  
The associatd Rammnujan equation has integer solution $ -31 $. 
And 
\begin{equation} \label{fmn_8_7} 
\sqrt[3]{\alpha} + \sqrt[3]{\beta} + \sqrt[3]{\gamma} = 
-\sqrt[3]{448} 
\end{equation}  
\begin{equation} \label{gmn_8_7} 
\frac{1}{\sqrt[3]{\alpha}} + \frac{1}{\sqrt[3]{\beta}} + \frac{1}{\sqrt[3]{\gamma}} = 
 \sqrt[3]{49} 
\end{equation}  
\end{theorem}

\begin{theorem} \label{tmn_8_10} 
Let 
$$
\alpha = \frac{(2\cos(2\theta)^8}{(2\cos(4\theta)^10},
\beta  = \frac{(2\cos(4\theta)^8}{(2\cos(8\theta)^10},
\gamma = \frac{(2\cos(8\theta)^8}{(2\cos(2\theta)^10}.
$$
Then $ \{ \alpha, \beta, \gamma \} $ are roots of the equation:
\begin{equation} \label{emn_8_10} 
x^3 - 2110 x^2 + 101 x^2 - 1 = 0
\end{equation}  
The associatd Rammnujan equation has integer solution $ 95 $. 
And 
\begin{equation} \label{fmn_8_10} 
\sqrt[3]{\alpha} + \sqrt[3]{\beta} + \sqrt[3]{\gamma} = 
\sqrt[3]{2401} 
\end{equation}  
\begin{equation} \label{gmn_8_10} 
\frac{1}{\sqrt[3]{\alpha}} + \frac{1}{\sqrt[3]{\beta}} + \frac{1}{\sqrt[3]{\gamma}} = 
 \sqrt[3]{392} 
\end{equation}  
\end{theorem}

\begin{theorem} \label{tmn_8_13} 
Let 
$$
\alpha = \frac{(2\cos(2\theta)^8}{(2\cos(4\theta)^13},
\beta  = \frac{(2\cos(4\theta)^8}{(2\cos(8\theta)^13},
\gamma = \frac{(2\cos(8\theta)^8}{(2\cos(2\theta)^13}.
$$
Then $ \{ \alpha, \beta, \gamma \} $ are roots of the equation:
\begin{equation} \label{emn_8_13} 
x^3 + 12345 x^2 - 298 x - 1 = 0
\end{equation}  
The associatd Rammnujan equation has integer solution $ 81 $. 
And 
\begin{equation} \label{fmn_8_13} 
\sqrt[3]{\alpha} + \sqrt[3]{\beta} + \sqrt[3]{\gamma} = 
-\sqrt[3]{12096} 
\end{equation}  
\begin{equation} \label{gmn_8_13} 
\frac{1}{\sqrt[3]{\alpha}} + \frac{1}{\sqrt[3]{\beta}} + \frac{1}{\sqrt[3]{\gamma}} = 
-\sqrt[3]{49} 
\end{equation}  
\end{theorem}

\begin{theorem} \label{tmn_8_16} 
Let 
$$
\alpha = \frac{(2\cos(2\theta)^8}{(2\cos(4\theta)^16},
\beta  = \frac{(2\cos(4\theta)^8}{(2\cos(8\theta)^16},
\gamma = \frac{(2\cos(8\theta)^8}{(2\cos(2\theta)^16}.
$$
Then $ \{ \alpha, \beta, \gamma \} $ are roots of the equation:
\begin{equation} \label{emn_8_16} 
x^3 - 72229 x^2 + 3818 x^2 - 1 = 0
\end{equation}  
The associatd Rammnujan equation has integer solution $ 767 $. 
And 
\begin{equation} \label{fmn_8_16} 
\sqrt[3]{\alpha} + \sqrt[3]{\beta} + \sqrt[3]{\gamma} = 
\sqrt[3]{74536} 
\end{equation}  
\begin{equation} \label{gmn_8_16} 
\frac{1}{\sqrt[3]{\alpha}} + \frac{1}{\sqrt[3]{\beta}} + \frac{1}{\sqrt[3]{\gamma}} = 
 \sqrt[3]{6125} 
\end{equation}  
\end{theorem}

\begin{theorem} \label{tmn_9_0} 
Let 
$$
\alpha = \frac{(2\cos(2\theta)^9}{(2\cos(4\theta)^0},
\beta  = \frac{(2\cos(4\theta)^9}{(2\cos(8\theta)^0},
\gamma = \frac{(2\cos(8\theta)^9}{(2\cos(2\theta)^0}.
$$
Then $ \{ \alpha, \beta, \gamma \} $ are roots of the equation:
\begin{equation} \label{emn_9_0} 
x^3 + 193 x^2 - 1460 x - 1 = 0
\end{equation}  
The associatd Rammnujan equation has integer solution $ 41 $. 
And 
\begin{equation} \label{fmn_9_0} 
\sqrt[3]{\alpha} + \sqrt[3]{\beta} + \sqrt[3]{\gamma} = 
-\sqrt[3]{64} 
\end{equation}  
\begin{equation} \label{gmn_9_0} 
\frac{1}{\sqrt[3]{\alpha}} + \frac{1}{\sqrt[3]{\beta}} + \frac{1}{\sqrt[3]{\gamma}} = 
-\sqrt[3]{1331} 
\end{equation}  
\end{theorem}

\begin{theorem} \label{tmn_9_3} 
Let 
$$
\alpha = \frac{(2\cos(2\theta)^9}{(2\cos(4\theta)^3},
\beta  = \frac{(2\cos(4\theta)^9}{(2\cos(8\theta)^3},
\gamma = \frac{(2\cos(8\theta)^9}{(2\cos(2\theta)^3}.
$$
Then $ \{ \alpha, \beta, \gamma \} $ are roots of the equation:
\begin{equation} \label{emn_9_3} 
x^3 + 25 x^2 - 753 x - 1 = 0
\end{equation}  
The associatd Rammnujan equation has integer solution $ 6 $. 
And 
\begin{equation} \label{fmn_9_3} 
\sqrt[3]{\alpha} + \sqrt[3]{\beta} + \sqrt[3]{\gamma} = 
-\sqrt[3]{1} 
\end{equation}  
\begin{equation} \label{gmn_9_3} 
\frac{1}{\sqrt[3]{\alpha}} + \frac{1}{\sqrt[3]{\beta}} + \frac{1}{\sqrt[3]{\gamma}} = 
-\sqrt[3]{729} 
\end{equation}  
\end{theorem}

\begin{theorem} \label{tmn_9_6} 
Let 
$$
\alpha = \frac{(2\cos(2\theta)^9}{(2\cos(4\theta)^6},
\beta  = \frac{(2\cos(4\theta)^9}{(2\cos(8\theta)^6},
\gamma = \frac{(2\cos(8\theta)^9}{(2\cos(2\theta)^6}.
$$
Then $ \{ \alpha, \beta, \gamma \} $ are roots of the equation:
\begin{equation} \label{emn_9_6} 
x^3 - 248 x^2 - 389 x - 1 = 0
\end{equation}  
The associatd Rammnujan equation has integer solution $ -43 $. 
And 
\begin{equation} \label{fmn_9_6} 
\sqrt[3]{\alpha} + \sqrt[3]{\beta} + \sqrt[3]{\gamma} = 
\sqrt[3]{125} 
\end{equation}  
\begin{equation} \label{gmn_9_6} 
\frac{1}{\sqrt[3]{\alpha}} + \frac{1}{\sqrt[3]{\beta}} + \frac{1}{\sqrt[3]{\gamma}} = 
-\sqrt[3]{512} 
\end{equation}  
\end{theorem}

\begin{theorem} \label{tmn_9_9} 
Let 
$$
\alpha = \frac{(2\cos(2\theta)^9}{(2\cos(4\theta)^9},
\beta  = \frac{(2\cos(4\theta)^9}{(2\cos(8\theta)^9},
\gamma = \frac{(2\cos(8\theta)^9}{(2\cos(2\theta)^9}.
$$
Then $ \{ \alpha, \beta, \gamma \} $ are roots of the equation:
\begin{equation} \label{emn_9_9} 
x^3 + 1460 x^2 - 193 x - 1 = 0
\end{equation}  
The associatd Rammnujan equation has integer solution $ 41 $. 
And 
\begin{equation} \label{fmn_9_9} 
\sqrt[3]{\alpha} + \sqrt[3]{\beta} + \sqrt[3]{\gamma} = 
-\sqrt[3]{1331} 
\end{equation}  
\begin{equation} \label{gmn_9_9} 
\frac{1}{\sqrt[3]{\alpha}} + \frac{1}{\sqrt[3]{\beta}} + \frac{1}{\sqrt[3]{\gamma}} = 
-\sqrt[3]{64} 
\end{equation}  
\end{theorem}

\begin{theorem} \label{tmn_9_12} 
Let 
$$
\alpha = \frac{(2\cos(2\theta)^9}{(2\cos(4\theta)^12},
\beta  = \frac{(2\cos(4\theta)^9}{(2\cos(8\theta)^12},
\gamma = \frac{(2\cos(8\theta)^9}{(2\cos(2\theta)^12}.
$$
Then $ \{ \alpha, \beta, \gamma \} $ are roots of the equation:
\begin{equation} \label{emn_9_12} 
x^3 - 8543 x^2 - 186 x - 1 = 0
\end{equation}  
The associatd Rammnujan equation has integer solution $ -183 $. 
And 
\begin{equation} \label{fmn_9_12} 
\sqrt[3]{\alpha} + \sqrt[3]{\beta} + \sqrt[3]{\gamma} = 
\sqrt[3]{8000} 
\end{equation}  
\begin{equation} \label{gmn_9_12} 
\frac{1}{\sqrt[3]{\alpha}} + \frac{1}{\sqrt[3]{\beta}} + \frac{1}{\sqrt[3]{\gamma}} = 
-\sqrt[3]{729} 
\end{equation}  
\end{theorem}

\begin{theorem} \label{tmn_9_15} 
Let 
$$
\alpha = \frac{(2\cos(2\theta)^9}{(2\cos(4\theta)^15},
\beta  = \frac{(2\cos(4\theta)^9}{(2\cos(8\theta)^15},
\gamma = \frac{(2\cos(8\theta)^9}{(2\cos(2\theta)^15}.
$$
Then $ \{ \alpha, \beta, \gamma \} $ are roots of the equation:
\begin{equation} \label{emn_9_15} 
x^3 + 49984 x^2 + 885 x^2 - 1 = 0
\end{equation}  
The associatd Rammnujan equation has integer solution $ -225 $. 
And 
\begin{equation} \label{fmn_9_15} 
\sqrt[3]{\alpha} + \sqrt[3]{\beta} + \sqrt[3]{\gamma} = 
-\sqrt[3]{50653} 
\end{equation}  
\begin{equation} \label{gmn_9_15} 
\frac{1}{\sqrt[3]{\alpha}} + \frac{1}{\sqrt[3]{\beta}} + \frac{1}{\sqrt[3]{\gamma}} = 
 \sqrt[3]{216} 
\end{equation}  
\end{theorem}

\begin{theorem} \label{tmn_10_2} 
Let 
$$
\alpha = \frac{(2\cos(2\theta)^10}{(2\cos(4\theta)^2},
\beta  = \frac{(2\cos(4\theta)^10}{(2\cos(8\theta)^2},
\gamma = \frac{(2\cos(8\theta)^10}{(2\cos(2\theta)^2}.
$$
Then $ \{ \alpha, \beta, \gamma \} $ are roots of the equation:
\begin{equation} \label{emn_10_2} 
x^3 - 101 x^2 + 2110 x^2 - 1 = 0
\end{equation}  
The associatd Rammnujan equation has integer solution $ 95 $. 
And 
\begin{equation} \label{fmn_10_2} 
\sqrt[3]{\alpha} + \sqrt[3]{\beta} + \sqrt[3]{\gamma} = 
\sqrt[3]{392} 
\end{equation}  
\begin{equation} \label{gmn_10_2} 
\frac{1}{\sqrt[3]{\alpha}} + \frac{1}{\sqrt[3]{\beta}} + \frac{1}{\sqrt[3]{\gamma}} = 
 \sqrt[3]{2401} 
\end{equation}  
\end{theorem}

\begin{theorem} \label{tmn_10_5} 
Let 
$$
\alpha = \frac{(2\cos(2\theta)^10}{(2\cos(4\theta)^5},
\beta  = \frac{(2\cos(4\theta)^10}{(2\cos(8\theta)^5},
\gamma = \frac{(2\cos(8\theta)^10}{(2\cos(2\theta)^5}.
$$
Then $ \{ \alpha, \beta, \gamma \} $ are roots of the equation:
\begin{equation} \label{emn_10_5} 
x^3 + 179 x^2 + 1088 x^2 - 1 = 0
\end{equation}  
The associatd Rammnujan equation has integer solution $ -73 $. 
And 
\begin{equation} \label{fmn_10_5} 
\sqrt[3]{\alpha} + \sqrt[3]{\beta} + \sqrt[3]{\gamma} = 
-\sqrt[3]{392} 
\end{equation}  
\begin{equation} \label{gmn_10_5} 
\frac{1}{\sqrt[3]{\alpha}} + \frac{1}{\sqrt[3]{\beta}} + \frac{1}{\sqrt[3]{\gamma}} = 
 \sqrt[3]{875} 
\end{equation}  
\end{theorem}

\begin{theorem} \label{tmn_10_8} 
Let 
$$
\alpha = \frac{(2\cos(2\theta)^10}{(2\cos(4\theta)^8},
\beta  = \frac{(2\cos(4\theta)^10}{(2\cos(8\theta)^8},
\gamma = \frac{(2\cos(8\theta)^10}{(2\cos(2\theta)^8}.
$$
Then $ \{ \alpha, \beta, \gamma \} $ are roots of the equation:
\begin{equation} \label{emn_10_8} 
x^3 - 1011 x^2 + 563 x^2 - 1 = 0
\end{equation}  
The associatd Rammnujan equation has integer solution $ 102 $. 
And 
\begin{equation} \label{fmn_10_8} 
\sqrt[3]{\alpha} + \sqrt[3]{\beta} + \sqrt[3]{\gamma} = 
\sqrt[3]{1323} 
\end{equation}  
\begin{equation} \label{gmn_10_8} 
\frac{1}{\sqrt[3]{\alpha}} + \frac{1}{\sqrt[3]{\beta}} + \frac{1}{\sqrt[3]{\gamma}} = 
 \sqrt[3]{875} 
\end{equation}  
\end{theorem}

\begin{theorem} \label{tmn_10_11} 
Let 
$$
\alpha = \frac{(2\cos(2\theta)^10}{(2\cos(4\theta)^11},
\beta  = \frac{(2\cos(4\theta)^10}{(2\cos(8\theta)^11},
\gamma = \frac{(2\cos(8\theta)^10}{(2\cos(2\theta)^11}.
$$
Then $ \{ \alpha, \beta, \gamma \} $ are roots of the equation:
\begin{equation} \label{emn_10_11} 
x^3 + 5912 x^2 + 269 x^2 - 1 = 0
\end{equation}  
The associatd Rammnujan equation has integer solution $ -73 $. 
And 
\begin{equation} \label{fmn_10_11} 
\sqrt[3]{\alpha} + \sqrt[3]{\beta} + \sqrt[3]{\gamma} = 
-\sqrt[3]{6125} 
\end{equation}  
\begin{equation} \label{gmn_10_11} 
\frac{1}{\sqrt[3]{\alpha}} + \frac{1}{\sqrt[3]{\beta}} + \frac{1}{\sqrt[3]{\gamma}} = 
 \sqrt[3]{56} 
\end{equation}  
\end{theorem}

\begin{theorem} \label{tmn_10_14} 
Let 
$$
\alpha = \frac{(2\cos(2\theta)^10}{(2\cos(4\theta)^14},
\beta  = \frac{(2\cos(4\theta)^10}{(2\cos(8\theta)^14},
\gamma = \frac{(2\cos(8\theta)^10}{(2\cos(2\theta)^14}.
$$
Then $ \{ \alpha, \beta, \gamma \} $ are roots of the equation:
\begin{equation} \label{emn_10_14} 
x^3 - 34590 x^2 + 381 x^2 - 1 = 0
\end{equation}  
The associatd Rammnujan equation has integer solution $ 375 $. 
And 
\begin{equation} \label{fmn_10_14} 
\sqrt[3]{\alpha} + \sqrt[3]{\beta} + \sqrt[3]{\gamma} = 
\sqrt[3]{35721} 
\end{equation}  
\begin{equation} \label{gmn_10_14} 
\frac{1}{\sqrt[3]{\alpha}} + \frac{1}{\sqrt[3]{\beta}} + \frac{1}{\sqrt[3]{\gamma}} = 
 \sqrt[3]{1512} 
\end{equation}  
\end{theorem}

\begin{theorem} \label{tmn_10_17} 
Let 
$$
\alpha = \frac{(2\cos(2\theta)^10}{(2\cos(4\theta)^17},
\beta  = \frac{(2\cos(4\theta)^10}{(2\cos(8\theta)^17},
\gamma = \frac{(2\cos(8\theta)^10}{(2\cos(2\theta)^17}.
$$
Then $ \{ \alpha, \beta, \gamma \} $ are roots of the equation:
\begin{equation} \label{emn_10_17} 
x^3 + 202381 x^2 - 2552 x - 1 = 0
\end{equation}  
The associatd Rammnujan equation has integer solution $ 557 $. 
And 
\begin{equation} \label{fmn_10_17} 
\sqrt[3]{\alpha} + \sqrt[3]{\beta} + \sqrt[3]{\gamma} = 
-\sqrt[3]{200704} 
\end{equation}  
\begin{equation} \label{gmn_10_17} 
\frac{1}{\sqrt[3]{\alpha}} + \frac{1}{\sqrt[3]{\beta}} + \frac{1}{\sqrt[3]{\gamma}} = 
-\sqrt[3]{875} 
\end{equation}  
\end{theorem}

\begin{theorem} \label{tmn_11_1} 
Let 
$$
\alpha = \frac{(2\cos(2\theta)^11}{(2\cos(4\theta)^1},
\beta  = \frac{(2\cos(4\theta)^11}{(2\cos(8\theta)^1},
\gamma = \frac{(2\cos(8\theta)^11}{(2\cos(2\theta)^1}.
$$
Then $ \{ \alpha, \beta, \gamma \} $ are roots of the equation:
\begin{equation} \label{emn_11_1} 
x^3 - 269 x^2 - 5912 x - 1 = 0
\end{equation}  
The associatd Rammnujan equation has integer solution $ -73 $. 
And 
\begin{equation} \label{fmn_11_1} 
\sqrt[3]{\alpha} + \sqrt[3]{\beta} + \sqrt[3]{\gamma} = 
\sqrt[3]{56} 
\end{equation}  
\begin{equation} \label{gmn_11_1} 
\frac{1}{\sqrt[3]{\alpha}} + \frac{1}{\sqrt[3]{\beta}} + \frac{1}{\sqrt[3]{\gamma}} = 
-\sqrt[3]{6125} 
\end{equation}  
\end{theorem}

\begin{theorem} \label{tmn_11_4} 
Let 
$$
\alpha = \frac{(2\cos(2\theta)^11}{(2\cos(4\theta)^4},
\beta  = \frac{(2\cos(4\theta)^11}{(2\cos(8\theta)^4},
\gamma = \frac{(2\cos(8\theta)^11}{(2\cos(2\theta)^4}.
$$
Then $ \{ \alpha, \beta, \gamma \} $ are roots of the equation:
\begin{equation} \label{emn_11_4} 
x^3 - 94 x^2 - 3049 x - 1 = 0
\end{equation}  
The associatd Rammnujan equation has integer solution $ -31 $. 
And 
\begin{equation} \label{fmn_11_4} 
\sqrt[3]{\alpha} + \sqrt[3]{\beta} + \sqrt[3]{\gamma} = 
\sqrt[3]{7} 
\end{equation}  
\begin{equation} \label{gmn_11_4} 
\frac{1}{\sqrt[3]{\alpha}} + \frac{1}{\sqrt[3]{\beta}} + \frac{1}{\sqrt[3]{\gamma}} = 
-\sqrt[3]{3136} 
\end{equation}  
\end{theorem}

\begin{theorem} \label{tmn_11_7} 
Let 
$$
\alpha = \frac{(2\cos(2\theta)^11}{(2\cos(4\theta)^7},
\beta  = \frac{(2\cos(4\theta)^11}{(2\cos(8\theta)^7},
\gamma = \frac{(2\cos(8\theta)^11}{(2\cos(2\theta)^7}.
$$
Then $ \{ \alpha, \beta, \gamma \} $ are roots of the equation:
\begin{equation} \label{emn_11_7} 
x^3 + 697 x^2 - 1572 x - 1 = 0
\end{equation}  
The associatd Rammnujan equation has integer solution $ 81 $. 
And 
\begin{equation} \label{fmn_11_7} 
\sqrt[3]{\alpha} + \sqrt[3]{\beta} + \sqrt[3]{\gamma} = 
-\sqrt[3]{448} 
\end{equation}  
\begin{equation} \label{gmn_11_7} 
\frac{1}{\sqrt[3]{\alpha}} + \frac{1}{\sqrt[3]{\beta}} + \frac{1}{\sqrt[3]{\gamma}} = 
-\sqrt[3]{1323} 
\end{equation}  
\end{theorem}

\begin{theorem} \label{tmn_11_10} 
Let 
$$
\alpha = \frac{(2\cos(2\theta)^11}{(2\cos(4\theta)^10},
\beta  = \frac{(2\cos(4\theta)^11}{(2\cos(8\theta)^10},
\gamma = \frac{(2\cos(8\theta)^11}{(2\cos(2\theta)^10}.
$$
Then $ \{ \alpha, \beta, \gamma \} $ are roots of the equation:
\begin{equation} \label{emn_11_10} 
x^3 - 4091 x^2 - 816 x - 1 = 0
\end{equation}  
The associatd Rammnujan equation has integer solution $ -171 $. 
And 
\begin{equation} \label{fmn_11_10} 
\sqrt[3]{\alpha} + \sqrt[3]{\beta} + \sqrt[3]{\gamma} = 
\sqrt[3]{3584} 
\end{equation}  
\begin{equation} \label{gmn_11_10} 
\frac{1}{\sqrt[3]{\alpha}} + \frac{1}{\sqrt[3]{\beta}} + \frac{1}{\sqrt[3]{\gamma}} = 
-\sqrt[3]{1323} 
\end{equation}  
\end{theorem}

\begin{theorem} \label{tmn_11_13} 
Let 
$$
\alpha = \frac{(2\cos(2\theta)^11}{(2\cos(4\theta)^13},
\beta  = \frac{(2\cos(4\theta)^11}{(2\cos(8\theta)^13},
\gamma = \frac{(2\cos(8\theta)^11}{(2\cos(2\theta)^13}.
$$
Then $ \{ \alpha, \beta, \gamma \} $ are roots of the equation:
\begin{equation} \label{emn_11_13} 
x^3 + 23937 x^2 - 361 x - 1 = 0
\end{equation}  
The associatd Rammnujan equation has integer solution $ 102 $. 
And 
\begin{equation} \label{fmn_11_13} 
\sqrt[3]{\alpha} + \sqrt[3]{\beta} + \sqrt[3]{\gamma} = 
-\sqrt[3]{23625} 
\end{equation}  
\begin{equation} \label{gmn_11_13} 
\frac{1}{\sqrt[3]{\alpha}} + \frac{1}{\sqrt[3]{\beta}} + \frac{1}{\sqrt[3]{\gamma}} = 
-\sqrt[3]{49} 
\end{equation}  
\end{theorem}

\begin{theorem} \label{tmn_11_16} 
Let 
$$
\alpha = \frac{(2\cos(2\theta)^11}{(2\cos(4\theta)^16},
\beta  = \frac{(2\cos(4\theta)^11}{(2\cos(8\theta)^16},
\gamma = \frac{(2\cos(8\theta)^11}{(2\cos(2\theta)^16}.
$$
Then $ \{ \alpha, \beta, \gamma \} $ are roots of the equation:
\begin{equation} \label{emn_11_16} 
x^3 - 140052 x^2 - 865 x - 1 = 0
\end{equation}  
The associatd Rammnujan equation has integer solution $ -759 $. 
And 
\begin{equation} \label{fmn_11_16} 
\sqrt[3]{\alpha} + \sqrt[3]{\beta} + \sqrt[3]{\gamma} = 
\sqrt[3]{137781} 
\end{equation}  
\begin{equation} \label{gmn_11_16} 
\frac{1}{\sqrt[3]{\alpha}} + \frac{1}{\sqrt[3]{\beta}} + \frac{1}{\sqrt[3]{\gamma}} = 
-\sqrt[3]{3136} 
\end{equation}  
\end{theorem}

\begin{theorem} \label{tmn_12_0} 
Let 
$$
\alpha = \frac{(2\cos(2\theta)^12}{(2\cos(4\theta)^0},
\beta  = \frac{(2\cos(4\theta)^12}{(2\cos(8\theta)^0},
\gamma = \frac{(2\cos(8\theta)^12}{(2\cos(2\theta)^0}.
$$
Then $ \{ \alpha, \beta, \gamma \} $ are roots of the equation:
\begin{equation} \label{emn_12_0} 
x^3 - 1186 x^2 + 16565 x^2 - 1 = 0
\end{equation}  
The associatd Rammnujan equation has integer solution $ 335 $. 
And 
\begin{equation} \label{fmn_12_0} 
\sqrt[3]{\alpha} + \sqrt[3]{\beta} + \sqrt[3]{\gamma} = 
\sqrt[3]{2197} 
\end{equation}  
\begin{equation} \label{gmn_12_0} 
\frac{1}{\sqrt[3]{\alpha}} + \frac{1}{\sqrt[3]{\beta}} + \frac{1}{\sqrt[3]{\gamma}} = 
 \sqrt[3]{17576} 
\end{equation}  
\end{theorem}

\begin{theorem} \label{tmn_12_3} 
Let 
$$
\alpha = \frac{(2\cos(2\theta)^12}{(2\cos(4\theta)^3},
\beta  = \frac{(2\cos(4\theta)^12}{(2\cos(8\theta)^3},
\gamma = \frac{(2\cos(8\theta)^12}{(2\cos(2\theta)^3}.
$$
Then $ \{ \alpha, \beta, \gamma \} $ are roots of the equation:
\begin{equation} \label{emn_12_3} 
x^3 + 186 x^2 + 8543 x^2 - 1 = 0
\end{equation}  
The associatd Rammnujan equation has integer solution $ -183 $. 
And 
\begin{equation} \label{fmn_12_3} 
\sqrt[3]{\alpha} + \sqrt[3]{\beta} + \sqrt[3]{\gamma} = 
-\sqrt[3]{729} 
\end{equation}  
\begin{equation} \label{gmn_12_3} 
\frac{1}{\sqrt[3]{\alpha}} + \frac{1}{\sqrt[3]{\beta}} + \frac{1}{\sqrt[3]{\gamma}} = 
 \sqrt[3]{8000} 
\end{equation}  
\end{theorem}

\begin{theorem} \label{tmn_12_6} 
Let 
$$
\alpha = \frac{(2\cos(2\theta)^12}{(2\cos(4\theta)^6},
\beta  = \frac{(2\cos(4\theta)^12}{(2\cos(8\theta)^6},
\gamma = \frac{(2\cos(8\theta)^12}{(2\cos(2\theta)^6}.
$$
Then $ \{ \alpha, \beta, \gamma \} $ are roots of the equation:
\begin{equation} \label{emn_12_6} 
x^3 - 493 x^2 + 4406 x^2 - 1 = 0
\end{equation}  
The associatd Rammnujan equation has integer solution $ 167 $. 
And 
\begin{equation} \label{fmn_12_6} 
\sqrt[3]{\alpha} + \sqrt[3]{\beta} + \sqrt[3]{\gamma} = 
\sqrt[3]{1000} 
\end{equation}  
\begin{equation} \label{gmn_12_6} 
\frac{1}{\sqrt[3]{\alpha}} + \frac{1}{\sqrt[3]{\beta}} + \frac{1}{\sqrt[3]{\gamma}} = 
 \sqrt[3]{4913} 
\end{equation}  
\end{theorem}

\begin{theorem} \label{tmn_12_9} 
Let 
$$
\alpha = \frac{(2\cos(2\theta)^12}{(2\cos(4\theta)^9},
\beta  = \frac{(2\cos(4\theta)^12}{(2\cos(8\theta)^9},
\gamma = \frac{(2\cos(8\theta)^12}{(2\cos(2\theta)^9}.
$$
Then $ \{ \alpha, \beta, \gamma \} $ are roots of the equation:
\begin{equation} \label{emn_12_9} 
x^3 + 2832 x^2 + 2271 x^2 - 1 = 0
\end{equation}  
The associatd Rammnujan equation has integer solution $ -183 $. 
And 
\begin{equation} \label{fmn_12_9} 
\sqrt[3]{\alpha} + \sqrt[3]{\beta} + \sqrt[3]{\gamma} = 
-\sqrt[3]{3375} 
\end{equation}  
\begin{equation} \label{gmn_12_9} 
\frac{1}{\sqrt[3]{\alpha}} + \frac{1}{\sqrt[3]{\beta}} + \frac{1}{\sqrt[3]{\gamma}} = 
 \sqrt[3]{1728} 
\end{equation}  
\end{theorem}

\begin{theorem} \label{tmn_12_12} 
Let 
$$
\alpha = \frac{(2\cos(2\theta)^12}{(2\cos(4\theta)^12},
\beta  = \frac{(2\cos(4\theta)^12}{(2\cos(8\theta)^12},
\gamma = \frac{(2\cos(8\theta)^12}{(2\cos(2\theta)^12}.
$$
Then $ \{ \alpha, \beta, \gamma \} $ are roots of the equation:
\begin{equation} \label{emn_12_12} 
x^3 - 16565 x^2 + 1186 x^2 - 1 = 0
\end{equation}  
The associatd Rammnujan equation has integer solution $ 335 $. 
And 
\begin{equation} \label{fmn_12_12} 
\sqrt[3]{\alpha} + \sqrt[3]{\beta} + \sqrt[3]{\gamma} = 
\sqrt[3]{17576} 
\end{equation}  
\begin{equation} \label{gmn_12_12} 
\frac{1}{\sqrt[3]{\alpha}} + \frac{1}{\sqrt[3]{\beta}} + \frac{1}{\sqrt[3]{\gamma}} = 
 \sqrt[3]{2197} 
\end{equation}  
\end{theorem}

\begin{theorem} \label{tmn_12_15} 
Let 
$$
\alpha = \frac{(2\cos(2\theta)^12}{(2\cos(4\theta)^15},
\beta  = \frac{(2\cos(4\theta)^12}{(2\cos(8\theta)^15},
\gamma = \frac{(2\cos(8\theta)^12}{(2\cos(2\theta)^15}.
$$
Then $ \{ \alpha, \beta, \gamma \} $ are roots of the equation:
\begin{equation} \label{emn_12_15} 
x^3 + 96919 x^2 + 444 x^2 - 1 = 0
\end{equation}  
The associatd Rammnujan equation has integer solution $ -141 $. 
And 
\begin{equation} \label{fmn_12_15} 
\sqrt[3]{\alpha} + \sqrt[3]{\beta} + \sqrt[3]{\gamma} = 
-\sqrt[3]{97336} 
\end{equation}  
\begin{equation} \label{gmn_12_15} 
\frac{1}{\sqrt[3]{\alpha}} + \frac{1}{\sqrt[3]{\beta}} + \frac{1}{\sqrt[3]{\gamma}} = 
 \sqrt[3]{27} 
\end{equation}  
\end{theorem}

\begin{theorem} \label{tmn_13_2} 
Let 
$$
\alpha = \frac{(2\cos(2\theta)^13}{(2\cos(4\theta)^2},
\beta  = \frac{(2\cos(4\theta)^13}{(2\cos(8\theta)^2},
\gamma = \frac{(2\cos(8\theta)^13}{(2\cos(2\theta)^2}.
$$
Then $ \{ \alpha, \beta, \gamma \} $ are roots of the equation:
\begin{equation} \label{emn_13_2} 
x^3 + 361 x^2 - 23937 x - 1 = 0
\end{equation}  
The associatd Rammnujan equation has integer solution $ 102 $. 
And 
\begin{equation} \label{fmn_13_2} 
\sqrt[3]{\alpha} + \sqrt[3]{\beta} + \sqrt[3]{\gamma} = 
-\sqrt[3]{49} 
\end{equation}  
\begin{equation} \label{gmn_13_2} 
\frac{1}{\sqrt[3]{\alpha}} + \frac{1}{\sqrt[3]{\beta}} + \frac{1}{\sqrt[3]{\gamma}} = 
-\sqrt[3]{23625} 
\end{equation}  
\end{theorem}

\begin{theorem} \label{tmn_13_5} 
Let 
$$
\alpha = \frac{(2\cos(2\theta)^13}{(2\cos(4\theta)^5},
\beta  = \frac{(2\cos(4\theta)^13}{(2\cos(8\theta)^5},
\gamma = \frac{(2\cos(8\theta)^13}{(2\cos(2\theta)^5}.
$$
Then $ \{ \alpha, \beta, \gamma \} $ are roots of the equation:
\begin{equation} \label{emn_13_5} 
x^3 + 298 x^2 - 12345 x - 1 = 0
\end{equation}  
The associatd Rammnujan equation has integer solution $ 81 $. 
And 
\begin{equation} \label{fmn_13_5} 
\sqrt[3]{\alpha} + \sqrt[3]{\beta} + \sqrt[3]{\gamma} = 
-\sqrt[3]{49} 
\end{equation}  
\begin{equation} \label{gmn_13_5} 
\frac{1}{\sqrt[3]{\alpha}} + \frac{1}{\sqrt[3]{\beta}} + \frac{1}{\sqrt[3]{\gamma}} = 
-\sqrt[3]{12096} 
\end{equation}  
\end{theorem}

\begin{theorem} \label{tmn_13_8} 
Let 
$$
\alpha = \frac{(2\cos(2\theta)^13}{(2\cos(4\theta)^8},
\beta  = \frac{(2\cos(4\theta)^13}{(2\cos(8\theta)^8},
\gamma = \frac{(2\cos(8\theta)^13}{(2\cos(2\theta)^8}.
$$
Then $ \{ \alpha, \beta, \gamma \} $ are roots of the equation:
\begin{equation} \label{emn_13_8} 
x^3 - 1956 x^2 - 6367 x - 1 = 0
\end{equation}  
The associatd Rammnujan equation has integer solution $ -213 $. 
And 
\begin{equation} \label{fmn_13_8} 
\sqrt[3]{\alpha} + \sqrt[3]{\beta} + \sqrt[3]{\gamma} = 
\sqrt[3]{1323} 
\end{equation}  
\begin{equation} \label{gmn_13_8} 
\frac{1}{\sqrt[3]{\alpha}} + \frac{1}{\sqrt[3]{\beta}} + \frac{1}{\sqrt[3]{\gamma}} = 
-\sqrt[3]{7000} 
\end{equation}  
\end{theorem}

\begin{theorem} \label{tmn_13_11} 
Let 
$$
\alpha = \frac{(2\cos(2\theta)^13}{(2\cos(4\theta)^11},
\beta  = \frac{(2\cos(4\theta)^13}{(2\cos(8\theta)^11},
\gamma = \frac{(2\cos(8\theta)^13}{(2\cos(2\theta)^11}.
$$
Then $ \{ \alpha, \beta, \gamma \} $ are roots of the equation:
\begin{equation} \label{emn_13_11} 
x^3 + 11463 x^2 - 3280 x - 1 = 0
\end{equation}  
The associatd Rammnujan equation has integer solution $ 291 $. 
And 
\begin{equation} \label{fmn_13_11} 
\sqrt[3]{\alpha} + \sqrt[3]{\beta} + \sqrt[3]{\gamma} = 
-\sqrt[3]{10584} 
\end{equation}  
\begin{equation} \label{gmn_13_11} 
\frac{1}{\sqrt[3]{\alpha}} + \frac{1}{\sqrt[3]{\beta}} + \frac{1}{\sqrt[3]{\gamma}} = 
-\sqrt[3]{2401} 
\end{equation}  
\end{theorem}

\begin{theorem} \label{tmn_13_14} 
Let 
$$
\alpha = \frac{(2\cos(2\theta)^13}{(2\cos(4\theta)^14},
\beta  = \frac{(2\cos(4\theta)^13}{(2\cos(8\theta)^14},
\gamma = \frac{(2\cos(8\theta)^13}{(2\cos(2\theta)^14}.
$$
Then $ \{ \alpha, \beta, \gamma \} $ are roots of the equation:
\begin{equation} \label{emn_13_14} 
x^3 - 67070 x^2 - 1733 x - 1 = 0
\end{equation}  
The associatd Rammnujan equation has integer solution $ -619 $. 
And 
\begin{equation} \label{fmn_13_14} 
\sqrt[3]{\alpha} + \sqrt[3]{\beta} + \sqrt[3]{\gamma} = 
\sqrt[3]{65219} 
\end{equation}  
\begin{equation} \label{gmn_13_14} 
\frac{1}{\sqrt[3]{\alpha}} + \frac{1}{\sqrt[3]{\beta}} + \frac{1}{\sqrt[3]{\gamma}} = 
-\sqrt[3]{3584} 
\end{equation}  
\end{theorem}

\begin{theorem} \label{tmn_13_17} 
Let 
$$
\alpha = \frac{(2\cos(2\theta)^13}{(2\cos(4\theta)^17},
\beta  = \frac{(2\cos(4\theta)^13}{(2\cos(8\theta)^17},
\gamma = \frac{(2\cos(8\theta)^13}{(2\cos(2\theta)^17}.
$$
Then $ \{ \alpha, \beta, \gamma \} $ are roots of the equation:
\begin{equation} \label{emn_13_17} 
x^3 + 392417 x^2 - 424 x - 1 = 0
\end{equation}  
The associatd Rammnujan equation has integer solution $ 137 $. 
And 
\begin{equation} \label{fmn_13_17} 
\sqrt[3]{\alpha} + \sqrt[3]{\beta} + \sqrt[3]{\gamma} = 
-\sqrt[3]{392000} 
\end{equation}  
\begin{equation} \label{gmn_13_17} 
\frac{1}{\sqrt[3]{\alpha}} + \frac{1}{\sqrt[3]{\beta}} + \frac{1}{\sqrt[3]{\gamma}} = 
-\sqrt[3]{7} 
\end{equation}  
\end{theorem}

\begin{theorem} \label{tmn_14_1} 
Let 
$$
\alpha = \frac{(2\cos(2\theta)^14}{(2\cos(4\theta)^1},
\beta  = \frac{(2\cos(4\theta)^14}{(2\cos(8\theta)^1},
\gamma = \frac{(2\cos(8\theta)^14}{(2\cos(2\theta)^1}.
$$
Then $ \{ \alpha, \beta, \gamma \} $ are roots of the equation:
\begin{equation} \label{emn_14_1} 
x^3 + 1733 x^2 + 67070 x^2 - 1 = 0
\end{equation}  
The associatd Rammnujan equation has integer solution $ -619 $. 
And 
\begin{equation} \label{fmn_14_1} 
\sqrt[3]{\alpha} + \sqrt[3]{\beta} + \sqrt[3]{\gamma} = 
-\sqrt[3]{3584} 
\end{equation}  
\begin{equation} \label{gmn_14_1} 
\frac{1}{\sqrt[3]{\alpha}} + \frac{1}{\sqrt[3]{\beta}} + \frac{1}{\sqrt[3]{\gamma}} = 
 \sqrt[3]{65219} 
\end{equation}  
\end{theorem}

\begin{theorem} \label{tmn_14_4} 
Let 
$$
\alpha = \frac{(2\cos(2\theta)^14}{(2\cos(4\theta)^4},
\beta  = \frac{(2\cos(4\theta)^14}{(2\cos(8\theta)^4},
\gamma = \frac{(2\cos(8\theta)^14}{(2\cos(2\theta)^4}.
$$
Then $ \{ \alpha, \beta, \gamma \} $ are roots of the equation:
\begin{equation} \label{emn_14_4} 
x^3 - 381 x^2 + 34590 x^2 - 1 = 0
\end{equation}  
The associatd Rammnujan equation has integer solution $ 375 $. 
And 
\begin{equation} \label{fmn_14_4} 
\sqrt[3]{\alpha} + \sqrt[3]{\beta} + \sqrt[3]{\gamma} = 
\sqrt[3]{1512} 
\end{equation}  
\begin{equation} \label{gmn_14_4} 
\frac{1}{\sqrt[3]{\alpha}} + \frac{1}{\sqrt[3]{\beta}} + \frac{1}{\sqrt[3]{\gamma}} = 
 \sqrt[3]{35721} 
\end{equation}  
\end{theorem}

\begin{theorem} \label{tmn_14_7} 
Let 
$$
\alpha = \frac{(2\cos(2\theta)^14}{(2\cos(4\theta)^7},
\beta  = \frac{(2\cos(4\theta)^14}{(2\cos(8\theta)^7},
\gamma = \frac{(2\cos(8\theta)^14}{(2\cos(2\theta)^7}.
$$
Then $ \{ \alpha, \beta, \gamma \} $ are roots of the equation:
\begin{equation} \label{emn_14_7} 
x^3 + 1369 x^2 + 17839 x^2 - 1 = 0
\end{equation}  
The associatd Rammnujan equation has integer solution $ -346 $. 
And 
\begin{equation} \label{fmn_14_7} 
\sqrt[3]{\alpha} + \sqrt[3]{\beta} + \sqrt[3]{\gamma} = 
-\sqrt[3]{2401} 
\end{equation}  
\begin{equation} \label{gmn_14_7} 
\frac{1}{\sqrt[3]{\alpha}} + \frac{1}{\sqrt[3]{\beta}} + \frac{1}{\sqrt[3]{\gamma}} = 
 \sqrt[3]{16807} 
\end{equation}  
\end{theorem}

\begin{theorem} \label{tmn_14_10} 
Let 
$$
\alpha = \frac{(2\cos(2\theta)^14}{(2\cos(4\theta)^10},
\beta  = \frac{(2\cos(4\theta)^14}{(2\cos(8\theta)^10},
\gamma = \frac{(2\cos(8\theta)^14}{(2\cos(2\theta)^10}.
$$
Then $ \{ \alpha, \beta, \gamma \} $ are roots of the equation:
\begin{equation} \label{emn_14_10} 
x^3 - 7934 x^2 + 9201 x^2 - 1 = 0
\end{equation}  
The associatd Rammnujan equation has integer solution $ 459 $. 
And 
\begin{equation} \label{fmn_14_10} 
\sqrt[3]{\alpha} + \sqrt[3]{\beta} + \sqrt[3]{\gamma} = 
\sqrt[3]{9317} 
\end{equation}  
\begin{equation} \label{gmn_14_10} 
\frac{1}{\sqrt[3]{\alpha}} + \frac{1}{\sqrt[3]{\beta}} + \frac{1}{\sqrt[3]{\gamma}} = 
 \sqrt[3]{10584} 
\end{equation}  
\end{theorem}

\begin{theorem} \label{tmn_14_13} 
Let 
$$
\alpha = \frac{(2\cos(2\theta)^14}{(2\cos(4\theta)^13},
\beta  = \frac{(2\cos(4\theta)^14}{(2\cos(8\theta)^13},
\gamma = \frac{(2\cos(8\theta)^14}{(2\cos(2\theta)^13}.
$$
Then $ \{ \alpha, \beta, \gamma \} $ are roots of the equation:
\begin{equation} \label{emn_14_13} 
x^3 + 46414 x^2 + 4735 x^2 - 1 = 0
\end{equation}  
The associatd Rammnujan equation has integer solution $ -535 $. 
And 
\begin{equation} \label{fmn_14_13} 
\sqrt[3]{\alpha} + \sqrt[3]{\beta} + \sqrt[3]{\gamma} = 
-\sqrt[3]{48013} 
\end{equation}  
\begin{equation} \label{gmn_14_13} 
\frac{1}{\sqrt[3]{\alpha}} + \frac{1}{\sqrt[3]{\beta}} + \frac{1}{\sqrt[3]{\gamma}} = 
 \sqrt[3]{3136} 
\end{equation}  
\end{theorem}

\begin{theorem} \label{tmn_14_19} 
Let 
$$
\alpha = \frac{(2\cos(2\theta)^14}{(2\cos(4\theta)^19},
\beta  = \frac{(2\cos(4\theta)^14}{(2\cos(8\theta)^19},
\gamma = \frac{(2\cos(8\theta)^14}{(2\cos(2\theta)^19}.
$$
Then $ \{ \alpha, \beta, \gamma \} $ are roots of the equation:
\begin{equation} \label{emn_14_19} 
x^3 + 1588863 x^2 + 3 x^2 - 1 = 0
\end{equation}  
The associatd Rammnujan equation has integer solution $ -3 $. 
And 
\begin{equation} \label{fmn_14_19} 
\sqrt[3]{\alpha} + \sqrt[3]{\beta} + \sqrt[3]{\gamma} = 
-\sqrt[3]{1588866} 
\end{equation}  
\begin{equation} \label{gmn_14_19} 
\frac{1}{\sqrt[3]{\alpha}} + \frac{1}{\sqrt[3]{\beta}} + \frac{1}{\sqrt[3]{\gamma}} = 
 \sqrt[3]{0} 
\end{equation}  
\end{theorem}

\begin{theorem} \label{tmn_15_0} 
Let 
$$
\alpha = \frac{(2\cos(2\theta)^15}{(2\cos(4\theta)^0},
\beta  = \frac{(2\cos(4\theta)^15}{(2\cos(8\theta)^0},
\gamma = \frac{(2\cos(8\theta)^15}{(2\cos(2\theta)^0}.
$$
Then $ \{ \alpha, \beta, \gamma \} $ are roots of the equation:
\begin{equation} \label{emn_15_0} 
x^3 + 6829 x^2 - 187926 x - 1 = 0
\end{equation}  
The associatd Rammnujan equation has integer solution $ 909 $. 
And 
\begin{equation} \label{fmn_15_0} 
\sqrt[3]{\alpha} + \sqrt[3]{\beta} + \sqrt[3]{\gamma} = 
-\sqrt[3]{4096} 
\end{equation}  
\begin{equation} \label{gmn_15_0} 
\frac{1}{\sqrt[3]{\alpha}} + \frac{1}{\sqrt[3]{\beta}} + \frac{1}{\sqrt[3]{\gamma}} = 
-\sqrt[3]{185193} 
\end{equation}  
\end{theorem}

\begin{theorem} \label{tmn_15_3} 
Let 
$$
\alpha = \frac{(2\cos(2\theta)^15}{(2\cos(4\theta)^3},
\beta  = \frac{(2\cos(4\theta)^15}{(2\cos(8\theta)^3},
\gamma = \frac{(2\cos(8\theta)^15}{(2\cos(2\theta)^3}.
$$
Then $ \{ \alpha, \beta, \gamma \} $ are roots of the equation:
\begin{equation} \label{emn_15_3} 
x^3 - 444 x^2 - 96919 x - 1 = 0
\end{equation}  
The associatd Rammnujan equation has integer solution $ -141 $. 
And 
\begin{equation} \label{fmn_15_3} 
\sqrt[3]{\alpha} + \sqrt[3]{\beta} + \sqrt[3]{\gamma} = 
\sqrt[3]{27} 
\end{equation}  
\begin{equation} \label{gmn_15_3} 
\frac{1}{\sqrt[3]{\alpha}} + \frac{1}{\sqrt[3]{\beta}} + \frac{1}{\sqrt[3]{\gamma}} = 
-\sqrt[3]{97336} 
\end{equation}  
\end{theorem}

\begin{theorem} \label{tmn_15_6} 
Let 
$$
\alpha = \frac{(2\cos(2\theta)^15}{(2\cos(4\theta)^6},
\beta  = \frac{(2\cos(4\theta)^15}{(2\cos(8\theta)^6},
\gamma = \frac{(2\cos(8\theta)^15}{(2\cos(2\theta)^6}.
$$
Then $ \{ \alpha, \beta, \gamma \} $ are roots of the equation:
\begin{equation} \label{emn_15_6} 
x^3 - 885 x^2 - 49984 x - 1 = 0
\end{equation}  
The associatd Rammnujan equation has integer solution $ -225 $. 
And 
\begin{equation} \label{fmn_15_6} 
\sqrt[3]{\alpha} + \sqrt[3]{\beta} + \sqrt[3]{\gamma} = 
\sqrt[3]{216} 
\end{equation}  
\begin{equation} \label{gmn_15_6} 
\frac{1}{\sqrt[3]{\alpha}} + \frac{1}{\sqrt[3]{\beta}} + \frac{1}{\sqrt[3]{\gamma}} = 
-\sqrt[3]{50653} 
\end{equation}  
\end{theorem}

\begin{theorem} \label{tmn_15_9} 
Let 
$$
\alpha = \frac{(2\cos(2\theta)^15}{(2\cos(4\theta)^9},
\beta  = \frac{(2\cos(4\theta)^15}{(2\cos(8\theta)^9},
\gamma = \frac{(2\cos(8\theta)^15}{(2\cos(2\theta)^9}.
$$
Then $ \{ \alpha, \beta, \gamma \} $ are roots of the equation:
\begin{equation} \label{emn_15_9} 
x^3 + 5485 x^2 - 25778 x - 1 = 0
\end{equation}  
The associatd Rammnujan equation has integer solution $ 461 $. 
And 
\begin{equation} \label{fmn_15_9} 
\sqrt[3]{\alpha} + \sqrt[3]{\beta} + \sqrt[3]{\gamma} = 
-\sqrt[3]{4096} 
\end{equation}  
\begin{equation} \label{gmn_15_9} 
\frac{1}{\sqrt[3]{\alpha}} + \frac{1}{\sqrt[3]{\beta}} + \frac{1}{\sqrt[3]{\gamma}} = 
-\sqrt[3]{24389} 
\end{equation}  
\end{theorem}

\begin{theorem} \label{tmn_15_12} 
Let 
$$
\alpha = \frac{(2\cos(2\theta)^15}{(2\cos(4\theta)^12},
\beta  = \frac{(2\cos(4\theta)^15}{(2\cos(8\theta)^12},
\gamma = \frac{(2\cos(8\theta)^15}{(2\cos(2\theta)^12}.
$$
Then $ \{ \alpha, \beta, \gamma \} $ are roots of the equation:
\begin{equation} \label{emn_15_12} 
x^3 - 32119 x^2 - 13297 x - 1 = 0
\end{equation}  
The associatd Rammnujan equation has integer solution $ -778 $. 
And 
\begin{equation} \label{fmn_15_12} 
\sqrt[3]{\alpha} + \sqrt[3]{\beta} + \sqrt[3]{\gamma} = 
\sqrt[3]{29791} 
\end{equation}  
\begin{equation} \label{gmn_15_12} 
\frac{1}{\sqrt[3]{\alpha}} + \frac{1}{\sqrt[3]{\beta}} + \frac{1}{\sqrt[3]{\gamma}} = 
-\sqrt[3]{15625} 
\end{equation}  
\end{